You are on page 1of 102

https://upscpdf.com/ https://upscpdf.com/ https://upscpdf.

com/

.
IASBABA’S PRELIMS CURRENT AFFAIRS TEST – MAY 2023

Q.1) Consider the following statements 1. Folate deficiency can cause severe
about the Pre-Conception and Pre-Natal birth defects.
Diagnostic Techniques (PCPNDT) Act of 2. Food fortification is a safe method
1994: of improving nutrition among
people.
1. It does not regulate the use of pre-
natal diagnostic techniques. Choose the correct code:
2. It does not allow any laboratory
centre or clinic to conduct any sex- a) 1 only
determining tests b) 2 only
including ultrasonography. c) Both 1 and 2
d) Neither 1 nor 2
3. Conducting any prenatal diagnostic
techniques in unregistered Q.4) Consider the following statements
facilities is an offense under the about the key features of Solid Waste
act. Management Rules, 2016:

Choose the correct code: 1. Waste collectors will have to pay


user fees for littering and non-
a) 1 and 2 only segregation.
b) 2 and 3 only 2. It does not cover sanitary waste like
c) 1 and 3 only diapers and sanitary pads.
d) 1, 2 and 3 3. The bio-degradable waste should
be processed and disposed of
Q.2) Consider the following statements through bio-methanation within
about bio-jet fuel: the premises.
4. All manufacturers of disposable
1. It is also known as sustainable
products shall provide necessary
aviation fuel (SAF).
financial assistance to local
2. It is produced using cooking
authorities.
oil and oil-rich seeds from plants.
3. It reduces greenhouse gas Choose the correct code:
emissions and enhances energy
security. a) 1 and 2 only
b) 2 and 3 only
Choose the correct code: c) 3 and 4 only
d) 1 and 4 only
a) 1 and 2 only
b) 2 and 3 only
Q.5) Consider the following statements
c) 1 and 3 only
about the ‘Command Cyber Operations
d) 1, 2 and 3
and Support Wings’ (CCOSWs):
Q.3) Consider the following statements:

https://upscpdf.com/ https://upscpdf.com/ https://upscpdf.com/


https://upscpdf.com/ https://upscpdf.com/ https://upscpdf.com/

.
IASBABA’S PRELIMS CURRENT AFFAIRS TEST – MAY 2023

1. These units are a specialized unit of 3. He wrote plays such as Manasi and
the Indian Navy. Sonar Tari.
2. It is responsible for safeguarding
the networks and enhancing the Choose the correct code:
cybersecurity posture of the Indian
a) 1 and 2 only
Army.
b) 2 and 3 only
Choose the correct code: c) 1 and 3 only
d) 1, 2 and 3
a) 1 only
b) 2 only Q.8) Consider the following statements
c) Both 1 and 2 about the ‘Geological Survey of India’ (GSI):
d) Neither 1 nor 2
1. It was set up primarily to find coal
Q.6) Consider the following statements deposits for the Railways in 1851.
about ‘Millets Experience Centre ‘(MEC): 2. It works under the Ministry of Earth
Sciences.
1. It is launched by the
Ministry of Agriculture & Farmers Choose the correct code:
Welfare.
a) 1 only
2. Visitors to the center can purchase
b) 2 only
a variety of ready-to-eat and ready-
c) Both 1 and 2
to-cook products from local millet
d) Neither 1 nor 2
start-ups.
3. The United Nations General
Q.9) Consider the following statements
Assembly has declared 2023 as
about the United Nations Economic and
the International Year of Millets.
Social Commission for Asia and the Pacific
Choose the correct code: (ESCAP):

a) 1 and 2 only 1. It promotes cooperation in pursuit


b) 2 and 3 only of solutions to sustainable
c) 1 and 3 only development challenges.
d) 1, 2 and 3 2. It is one of the five regional
commissions of the United Nations.
Q.7) Consider the following statements 3. It reports to the United Nations
about Rabindranath Tagore: Security Council.

1. He welcomed western science and Choose the correct code:


western beliefs.
a) 1 only
2. In his view, traditional schools
b) 2 and 3 only
imprison children.
c) 3 only

https://upscpdf.com/ https://upscpdf.com/ https://upscpdf.com/


https://upscpdf.com/ https://upscpdf.com/ https://upscpdf.com/

.
IASBABA’S PRELIMS CURRENT AFFAIRS TEST – MAY 2023

d) 1 and 2 only 2. It organises India’s biggest trade


show IIJS Premiere.
Q.10) Consider the following statements 3. It is the nodal agency to execute
about the Battle of Haldighati: Kimberly Process Certification
Scheme in India.
1. It was between Rana Pratap Singh
of Mewar and Raja Man Singh of Choose the correct code:
Amber.
2. In this battle, Mughal forces were a) 1 and 2 only
defeated by Maharana Pratap. b) 2 and 3 only
c) 1 and 3 only
Choose the correct code: d) 1, 2 and 3

a) 1 only
b) 2 only Q.13) Logistic Performance Index is
c) Both 1 and 2 published by?
d) Neither 1 nor 2
a) World Bank
b) IMF
Q.11) Consider the following statements
c) United Nations Development
about Coco Islands:
Programme
1. They are a small group of islands d) ILO
located in the Arabian Sea.
2. They are geologically an extended Q.14) Consider the following statements
division of the Arakan Mountains. about Saint Samarth Ramdas:
3. It was a self-governing crown
colony even after Burma was 1. He served as an inspiration for Bal
separated from British India. Gangadhar Tilak, Rajwade, Keshav
Hedgewar, and Ramchandra
Choose the correct code: Ranade.
2. His literary works
a) 1 and 2 only
include Karunashtakas, Dasbodh,
b) 2 and 3 only
Yuddhakand, Sunderkand,
c) 1 and 3 only
Poorvarambh, and Antarbhav.
d) 1, 2 and 3
3. His writings include strong
expressions encouraging
Q.12) Consider the following statements
nationalism to counter aggressive
about the Gems and Jewellery Export
Muslim invaders.
Promotion Council (GJEPC):
Choose the correct code:
1. It drives India’s export-led growth
in the gem and jewellery sector. a) 1 and 2 only
b) 2 and 3 only

https://upscpdf.com/ https://upscpdf.com/ https://upscpdf.com/


https://upscpdf.com/ https://upscpdf.com/ https://upscpdf.com/

.
IASBABA’S PRELIMS CURRENT AFFAIRS TEST – MAY 2023

c) 1 and 3 only 2. It is the brightest star in the


d) 1, 2 and 3 southern constellation Piscis
Austrinus.
Q.15) Consider the following statements 3. It is used in navigation.
about Gold Rush:
Choose the correct code:
1. It is the phenomenon of adding
gold to the government’s reserves a) 1 and 2 only
at an unusual pace. b) 2 and 3 only
2. Its objective is to safeguard returns c) 1 and 3 only
on gold amid global uncertainty d) 1, 2 and 3
and a rising inflation scenario.
Q.18) Consider the following statements
Choose the correct code: about the ‘Central Board of Indirect Taxes
and Customs’ (CBIC):
a) 1 only
b) 2 only 1. It is a part of the Department of
c) Both 1 and 2 Revenue under the Ministry of
d) Neither 1 nor 2 Finance.
2. It is a statutory body established
Q.16) Recently Pala Lake was in news. under the Central Boards of
Consider the following statements about Revenue Act, of 1963.
Palak Lake: 3. It is the nodal national
agency responsible for
1. It is the largest natural lake in administering customs and GST.
Sikkim.
2. It falls under the Indo-Burma Choose the correct code:
biodiversity hotspot.
a) 1 and 2 only
Choose the correct code: b) 2 and 3 only
c) 1 and 3 only
a) 1 only d) 1, 2 and 3
b) 2 only
c) Both 1 and 2 Q.19) Consider the following statements
d) Neither 1 nor 2 about the ‘Directorate General of Civil
Aviation’ (DGCA):
Q.17) Consider the following statements
about Fomalhaut: 1. It is a non-statutory body that
comes under the Ministry of Civil
1. It is a comet discovered in visible Aviation.
light by NASA. 2. It is responsible for the
registration of civil aircraft.
3. It is headquartered in New Delhi.

https://upscpdf.com/ https://upscpdf.com/ https://upscpdf.com/


https://upscpdf.com/ https://upscpdf.com/ https://upscpdf.com/

.
IASBABA’S PRELIMS CURRENT AFFAIRS TEST – MAY 2023

Choose the correct code: b) 2 only


c) Both 1 and 2
a) 2 and 3 only d) Neither 1 nor 2
b) 2 only
c) 3 only Q.23) Consider the following statements
d) 1 and 3 only about ‘Hydrogen Sulphide’ (H2S):

Q.20) PRET is an initiative of which 1. It is a colourless chalcogen hydride


organization? gas with a characteristic foul
a) World Bank odour.
b) UNEP 2. It does not dissolve in water and oil.
c) FAO 3. It is used to produce heavy
d) WHO water for nuclear power plants.

Q.21) Consider the following statements Choose the correct code:


about the ‘Visva-Bharati University’:
a) 1 and 2 only
1. It was set up by M.K. Gandhi. b) 2 and 3 only
2. He did not have a good opinion c) 1 and 3 only
about the Western method of d) 1, 2 and 3
education introduced by the British
in India Q.24) Consider the following statements
3. It followed open-air education. about ‘Poshan Bhi, Padhai Bhi Scheme’:

Choose the correct code: 1. It is launched by the Ministry of


Education.
a) 1 and 2 only 2. It emphasizes the links with
b) 2 and 3 only primary education as well as early
c) 3 only childhood health and nutrition
d) 1, 2 and 3 services.

Q.22) Consider the following statements Choose the correct code:


about Monlam Chenmo:
a) 1 only
1. It is an annual five-day-long Great b) 2 only
Prayer Festival of Ladakh. c) Both 1 and 2
2. It is a mass prayer camp by Jain d) Neither 1 nor 2
monks and nuns for world peace
and happiness. Q.25) Consider the following statements
about the iDrone initiative:
Choose the correct code:

a) 1 only

https://upscpdf.com/ https://upscpdf.com/ https://upscpdf.com/


https://upscpdf.com/ https://upscpdf.com/ https://upscpdf.com/

.
IASBABA’S PRELIMS CURRENT AFFAIRS TEST – MAY 2023

1. It aims to transport essential 2. They are part of the country


medical resources to remote areas of Ecuador, in South America.
within the country.
2. It is an initiative of the Ministry of Choose the correct code:
Civil Aviation.
a) 1 only
Choose the correct code: b) 2 only
c) Both 1 and 2
a) 1 only d) Neither 1 nor 2
b) 2 only
c) Both 1 and 2 Q.29) Consider the following statements
d) Neither 1 nor 2 about the Arab League:

Q.26) Which Indian state has launched the 1. It was formally known as
Gajapati Irrigation Project? the League of Arab States and was
established in 1975.
a) Karnataka 2. Its founding members include
b) Jharkhand Egypt, Syria, Lebanon, Iraq, and
c) Andhra Pradesh Saudi Arabia.
d) Odisha
Choose the correct code:
Q.27) Consider the following statements
a) 1 only
about the ‘Central Electricity Authority’:
b) 2 only
1. It is a statutory organization c) Both 1 and 2
constituted under the Electricity d) Neither 1 nor 2
(Supply) Act, 1948.
2. It advises the government on Q.30) Consider the following statements
matters relating to the National about Scary Barbie:
Electricity Policy (NEP).
1. It is a supermassive exoplanet.
Choose the correct code: 2. It was discovered using an AI
engine called REFITT.
a) 1 only
b) 2 only Choose the correct code:
c) Both 1 and 2
a) 1 only
d) Neither 1 nor 2
b) 2 only
c) Both 1 and 2
Q.28) Consider the following statements
d) Neither 1 nor 2
about the Galápagos Islands:

1. They are a chain of islands in the Q.31) Donanemab is a drug developed


Atlantic Ocean. for the treatment of?

https://upscpdf.com/ https://upscpdf.com/ https://upscpdf.com/


https://upscpdf.com/ https://upscpdf.com/ https://upscpdf.com/

https://upscpdf.com/ https://upscpdf.com/ https://upscpdf.com/


https://upscpdf.com/ https://upscpdf.com/ https://upscpdf.com/

https://upscpdf.com/ https://upscpdf.com/ https://upscpdf.com/


https://upscpdf.com/ https://upscpdf.com/ https://upscpdf.com/

https://upscpdf.com/ https://upscpdf.com/ https://upscpdf.com/


https://upscpdf.com/ https://upscpdf.com/ https://upscpdf.com/

.
IASBABA’S PRELIMS CURRENT AFFAIRS TEST – MAY 2023

Q.46) In the context of ‘Anji Khad Bridge’ 2. The park is named after a temple
which of the statements below are true: known as Kalesar Mahadev
Temple.
1. It will be the first cable-stayed 3. Simbalbara National Park nested in
railway bridge in India. Shivalik Hills lies to its north and
2. Anji River is the tributary of the shares a border with Himachal
Jhelum River. Pradesh.
3. It will connect Ladakh with the rest
Which of the above statement/s is/are
of India.
correct?
Which of the above statement/s is/are
a) 1 and 3 only
correct?
b) 1 and 2 only
a) 1 and 2 only c) 2 only
b) 2 and 3 only d) 1, 2 and 3 only
c) 1 only
d) 1, and 3 only
Q.49) Consider the following statements
about the King Cobra:
Q.47) With respect to ‘Lesser Flamingos’
consider the following statements: 1. They are the only snakes in the
world that build nests for their
1. It is the largest of all flamingos but
eggs.
has the smallest population.
2. They live in the rainforests and
2. Their diet consists entirely of
plains of India, southern China, and
microscopic blue-green algae and
Southeast Asia.
benthic diatoms found only in
3. They are listed as endangered on
alkaline salt lakes, salt pans,
the IUCN Red List.
estuaries, and salt lagoons.
3. Their IUCN status is ‘Vulnerable.’ Choose the correct code:
Select the incorrect answer using the codes
a) 1 and 2 only
given below:
b) 2 and 3 only
a) 1 and 2 only c) 1 and 3 only
b) 1 and 3 only d) 1, 2 and 3
c) 2 and 3 only
d) 1, 2 and 3 Q.50) Consider the following statements
about the Mridangam:

Q.48) Consider the following statement 1. It is the classical drum of South


with respect to Kalesar National Park. Indian music.
1. It is in the Yamnunagar district of 2. It is one of the oldest Indian
Haryana. percussion instruments.
3. It is made of jackwood.

https://upscpdf.com/ https://upscpdf.com/ https://upscpdf.com/


https://upscpdf.com/ https://upscpdf.com/ https://upscpdf.com/

.
IASBABA’S PRELIMS CURRENT AFFAIRS TEST – MAY 2023

Choose the correct code: Q.53) Consider the following statements


about Sea Butterflies:
a) 1 only
b) 2 only 1. They are a suborder of sea frogs.
c) 1 and 2 only 2. They are holoplanktonic.
d) 1, 2 and 3 3. They are found in all oceans.
4. They help in transporting carbon
Q.51) Consider the following statements from the surface to the deep ocean.
regarding the Fisheries Sector in India:
Choose the correct code:
1. India is the 4th largest exporter of
fish in the world. a) 1, 2 and 4
2. India ranks 3rd in inland capture b) 2 and 4 only
fish production in the world. c) 2, 3 and 4
3. Sagar Parikrama is a navigation d) 1, 2, 3 and 4
journey to be conducted in all
states/UTs. Q.54) Consider the following statements
about Artificial Sweeteners:
Choose the correct code:
1. They provide a sweet taste without
a) 1 and 2 the high-calorie content of regular
b) 1 only sugar.
c) 2 and 3 2. Saccharin, aspartame, and
d) 3 only acesulfame are examples of
artificial sweeteners.
Q.52) Consider the following statements 3. Its usage leads to diabetes and
regarding the Global Report on Internal tooth decay.
Displacement 2023 (GRID-2023):
Choose the correct code:
1. Afghanistan had the highest
number of disaster displacements a) 1 and 2
in the world in 2022. b) 2 and 3
2. The number of people living in c) 1 and 3
internal displacement reached a d) 1, 2 and 3
record low in 2022.
Q.55) Consider the following statements
Choose the correct code: about iDEX:

a) 1 only 1. It is an ecosystem to foster


b) 2 only innovation and technology
c) Both 1 and 2 development in nuclear energy.
d) Neither 1 nor 2 2. It provides funding/grants to Micro
Small and Medium Enterprises

https://upscpdf.com/ https://upscpdf.com/ https://upscpdf.com/


https://upscpdf.com/ https://upscpdf.com/ https://upscpdf.com/

.
IASBABA’S PRELIMS CURRENT AFFAIRS TEST – MAY 2023

(MSMEs), start-ups, and individual 2. The theme of its 6th edition was
innovators. "Peace Prosperity and Partnership
3. It is funded and managed for a Resilient Future".
by Defence Innovation
Organisation (DIO). Choose the correct code:

Choose the correct code: a) 1 only


b) 2 only
a) 1 and 2 c) Both 1 and 2
b) 2 and 3 d) Neither 1 nor 2
c) 1 and 3
d) 1, 2 and 3 Q.58) Consider the following statements
about Land Reclamation:
Q.56) Consider the following statements
about the Aadhaar-enabled Payment 1. It refers to the process of creating
System (AePS): new land by altering the
topography of existing bodies of
1. It is a bank-led model that water.
allows online interoperable 2. It leads to the loss of wetlands,
financial transactions at micro- coastal flooding, and the
ATMs only. destruction of the seabed
2. It helps in deepening social security ecosystem.
by facilitating cash transfers from
various government schemes. Choose the correct code:
3. It eliminates the need for OTPs,
a) 1 only
bank account details, and other
b) 2 only
financial information.
c) Both 1 and 2
Choose the correct code: d) Neither 1 nor 2

a) 1 and 2 Q.59) Consider the following statements


b) 2 and 3 about Transformers in Machine Learning:
c) 1 and 3
d) 1, 2 and 3 1. They are a type of deep learning
model used for natural language
Q.57) Consider the following statements processing (NLP) and computer
about the Indian Ocean Conference: vision (CV) tasks.
2. They consist of an encoder and a
1. It is a flagship conference organized decoder, which work together to
by the Association of Southeast process input and generate output.
Asian Nations (ASEAN). 3. They have applications in language
processing and image
understanding.

https://upscpdf.com/ https://upscpdf.com/ https://upscpdf.com/


https://upscpdf.com/ https://upscpdf.com/ https://upscpdf.com/

.
IASBABA’S PRELIMS CURRENT AFFAIRS TEST – MAY 2023

Choose the correct code: 1. It is a tool for the formal


maintenance of the local
a) 1 and 2 knowledge of biodiversity.
b) 2 and 3 2. It creates awareness among the
c) 1 and 3 people about biodiversity
d) 1, 2 and 3 conservation and sustainable
utilization.

Q.60) Which of the following disease is Choose the correct code:


treated by using Donanemab and
Lecanemab drugs? a) 1 only
b) 2 only
a) Leprosy c) Both 1 and 2
b) Alzheimer’s d) Neither 1 nor 2
c) Epilepsy
d) Malaria Q.63) Consider the following statements:

1. The International Museum Expo


Q.61) Consider the following statements 2023 Mascot was inspired by the
regarding different types of Hysterectomy: craft of Channapatna toys.
2. Dancing Girl is a prehistoric bronze
1. When only the uterus is removed, it
sculpture made in the lost wax
is called a partial hysterectomy.
casting technique.
2. When the uterus and cervix are
removed, it is called a radical Choose the correct code:
hysterectomy.
3. When the uterus, cervix, part of the a) 1 only
vagina, and a wide area of b) 2 only
ligaments and tissues around these c) Both 1 and 2
organs are removed, it is called d) Neither 1 nor 2
a total hysterectomy.
Q.64) Consider the following statements
Choose the correct code: about the Sengol:

a) 1 only 1. It originated from the Gupta


b) 2 only dynasty.
c) 3 only 2. It symbolises power transfer from
d) None one king to his successor.
Choose the correct code:
Q.62) Consider the following statements a) 1 only
about the People’s Biodiversity Register: b) 2 only
c) Both 1 and 2
d) Neither 1 nor 2

https://upscpdf.com/ https://upscpdf.com/ https://upscpdf.com/


https://upscpdf.com/ https://upscpdf.com/ https://upscpdf.com/

.
IASBABA’S PRELIMS CURRENT AFFAIRS TEST – MAY 2023

Q.65) Consider the following statements Q.68) Consider the following statements
about Forum Shopping: about Small Modular Reactors (SMRs):

1. It is the strategy of some 1. They are advanced nuclear reactors


lawyers who try to find the most with a power capacity of up to 300
favourable court for their case. MW (e) per unit.
2. It is defined in the Constitution of 2. They can be sited in locations not
India and the Indian statute. suitable for larger nuclear power
plants.
Choose the correct code: 3. They can be designed to operate
for up to 30 years without
a) 1 only
refuelling.
b) 2 only
c) Both 1 and 2 Choose the correct code:
d) Neither 1 nor 2
a) 1 and 2
Q.66) India has the Migration and Mobility b) 2 and 3
Partnership Agreement with which of the c) 1 and 3
following countries? d) 1, 2 and 3

a) Finland Q.69) Consider the following statements


b) Australia about the Insurance Information Bureau of
c) France India (IIB):
d) All of the above
1. It is a constitutional body
Q.67) Consider the following statements: established under the Insurance
Regulatory and Development
1. Article 123 of the Constitution of Authority of India (IRDAI).
India grants power to the 2. It facilitates data sharing and
Parliament to promulgate exchange among insurers,
ordinances at the centre. intermediaries, and regulators in an
2. Article 213 of the Constitution of insurance ecosystem.
India grants power to the Governor 3. It handles the Central Index
to promulgate ordinances in the Server and helps in the de-
state. duplication of demat accounts.

Choose the correct code: Choose the correct code:

a) 1 only a) 1 and 2
b) 2 only b) 2 and 3
c) Both 1 and 2 c) 1 and 3
d) Neither 1 nor 2 d) 1, 2 and 3

https://upscpdf.com/ https://upscpdf.com/ https://upscpdf.com/


https://upscpdf.com/ https://upscpdf.com/ https://upscpdf.com/

.
IASBABA’S PRELIMS CURRENT AFFAIRS TEST – MAY 2023

Q.73) Consider the following statements


Q.70) Consider the following statements: about Forum for India–Pacific Islands
Cooperation (FIPIC ) Summit:
1. INS Sindhuratna is a Sindhughosh-
class solar-electric submarine of 1. It was established by India as a part
the Indian Navy. of the Act East Policy.
2. Sindhughosh-class submarines 2. It consists of the Cook Islands, Fiji,
are Kilo-class diesel-electric Kiribati, and Marshall Islands.
submarines.
Choose the correct code:
Choose the correct code:
a) 1 only
a) 1 only b) 2 only
b) 2 only c) Both 1 and 2
c) Both 1 and 2 d) Neither 1 nor 2
d) Neither 1 nor 2
Q.74) Consider the following statements
Q.71) Consider the following statements about Koundinya Wildlife Sanctuary:
regarding Baralacha La Pass:
1. It is the only sanctuary in Tamil
1. It is a high mountain pass in the Nadu with a population of Asian
Zanskar range of Uttarakhand. elephants.
2. Chandra and Bhaga rivers originate 2. It has tropical dry deciduous forest,
near the Baralacha La Pass. with patches of thorn, scrub, and
grassy plains.
Choose the correct code:
Choose the correct code:
a) 1 only
b) 2 only a) 1 only
c) Both 1 and 2 b) 2 only
d) Neither 1 nor 2 c) Both 1 and 2
d) Neither 1 nor 2
Q.72) Which of the following is NOT a
member of G7? Q.75) Consider the following statements
about Deputy Chief Minister:
a) United States
b) Germany 1. Article 163 and Article 164 mention
c) Italy a Deputy Chief Minister.
d) India 2. He/She enjoys the same pay and
perks as a Chief Minister.

Choose the correct code:

a) 1 only

https://upscpdf.com/ https://upscpdf.com/ https://upscpdf.com/


https://upscpdf.com/ https://upscpdf.com/ https://upscpdf.com/

.
IASBABA’S PRELIMS CURRENT AFFAIRS TEST – MAY 2023

b) 2 only 2. It is a network only of governments


c) Both 1 and 2 for collecting and analyzing
d) Neither 1 nor 2 pathogen samples.

Q.76) Consider the following statements: Choose the correct code:

1. Asian elephants are the largest land a) 1 only


animals on Earth. b) 2 only
2. Elephants are matriarchal in c) Both 1 and 2
nature. d) Neither 1 nor 2
3. The Asian elephant is listed as
endangered on the IUCN Red List. Q.79) Consider the following statements
about the Corporate Debt Market
Choose the correct code: Development Fund:

a) 1 and 2 1. It is a form of an Alternative


b) 2 and 3 Investment Fund (AIF) with a
c) 1 and 3 corpus of ₹30,000 crore.
d) 1, 2 and 3
2. It will be a backstop facility for the
Q.77) Consider the following statements purchase of investment-grade
about Ectothermy: corporate debt securities in times
of stress.
1. It is the state of having a stable Choose the correct code:
body temperature without getting
affected by the environment. a) 1 only
2. It distinguishes fishes, amphibians, b) 2 only
and reptiles from birds and c) Both 1 and 2
mammals. d) Neither 1 nor 2

Choose the correct code: Q.80) Consider the following statements


about the Institute of Chartered
a) 1 only Accountants of India:
b) 2 only
c) Both 1 and 2 1. It is the only licensing cum
d) Neither 1 nor 2 regulating body of the financial
audit and accountancy profession
Q.78) Consider the following statements in India.
about the International Pathogen 2. It is a statutory body established
Surveillance Network (IPSN): under the Chartered Accountants
Act, of 1949.
1. It is a global network launched by
the World Health Organization.

https://upscpdf.com/ https://upscpdf.com/ https://upscpdf.com/


https://upscpdf.com/ https://upscpdf.com/ https://upscpdf.com/

.
IASBABA’S PRELIMS CURRENT AFFAIRS TEST – MAY 2023

3. It recommends the accounting 4. Gelugpa was founded by Gonchok


standards to be followed by Gyelpo.
companies in India.
Choose the correct code:
Choose the correct code:
a) 1 and 2
a) 1 and 2 b) 3 and 4
b) 2 and 3 c) 2 and 3
c) 1 and 3 d) 1 and 4
d) 1, 2 and 3
Q.83) Consider the following statements
Q.81) Consider the following statements: about the National AYUSH Mission:

1. Hydrogen produced from fossil 1. It is a centrally sponsored scheme


fuels is called brown hydrogen. under the Ministry of Health and
2. Hydrogen produced from Family Welfare.
coal where the emissions are 2. The central government provides a
released into the air is called grey 60% share and the state
hydrogen. government provides a 40% share
3. Hydrogen produced entirely from for its funding.
renewable power sources is
called green hydrogen. Choose the correct code:
4. Hydrogen produced from fossil
a) 1 only
fuels with carbon capture and
b) 2 only
storage options is called blue
c) Both 1 and 2
hydrogen.
d) Neither 1 nor 2
Choose the correct code:
Q.84) Consider the following statements
a) One statement is correct about Green Deposits:
b) Two statements are correct
c) Three statements are correct 1. They are interest-bearing deposits
d) Four statements are correct offered by both banks and NBFCs.
2. Small Finance Banks are not eligible
Q.82) In the context of Tibetan Buddhist for it.
groups, consider the following statements: 3. They are accepted in Indian Rupees
denominations only.
1. Nyingmapa was founded by
Padmasambhava. Choose the correct code:
2. Kagyupa was founded by Tilopa.
a) 1 and 2
3. Sakyapa was founded by Tsong
b) 2 and 3
Khapa Lobsang Drakpa.
c) 1 and 3

https://upscpdf.com/ https://upscpdf.com/ https://upscpdf.com/


https://upscpdf.com/ https://upscpdf.com/ https://upscpdf.com/

.
IASBABA’S PRELIMS CURRENT AFFAIRS TEST – MAY 2023

d) 1, 2 and 3 2. It is launched by National


Investigation Agency across all the
Q.85) Consider the following statements states.
about the Indian Renewable Energy
Development Agency Limited (IREDA): Choose the correct code:

1. It is a Public Limited Government a) 1 only


Company established as a Non- b) 2 only
Banking Financial Institution. c) Both 1 and 2
2. It is under the administrative d) Neither 1 nor 2
control of the Ministry of New and
Renewable Energy (MNRE). Q.88) Consider the following statements
about the European Bank for
Choose the correct code: Reconstruction and Development (EBRD):

a) 1 only 1. It is a multilateral financial


b) 2 only institution that is publicly owned by
c) Both 1 and 2 shareholders in 69 countries.
d) Neither 1 nor 2 2. It is headquartered in Paris, France.
3. It only supports countries that are
Q.86) Consider the following statements committed to democratic
about the Global Depository Receipts: principles.

1. It is a type of bank certificate that Choose the correct code:


represents shares of stock in the
International Monetary Fund. a) 1 and 2
2. They are used by issuers to raise b) 2 and 3
capital from international investors c) 1 and 3
through private placement. d) 1, 2 and 3

Choose the correct code: Q.89) Consider the following:


National Parks/Wildlife State
a) 1 only Sanctuaries
b) 2 only 1. Kibber Wildlife Himachal
c) Both 1 and 2 Sanctuary Pradesh
d) Neither 1 nor 2
2. Ulley Valley Ladakh

Q.87) Consider the following statements 3. Hemis National Himachal


about Operation Dhvast: Park Pradesh

1. It focuses on the terrorist-gangster- 4. Kishtwar Jammu and


drug smugglers’ network cases. National Park Kashmir

https://upscpdf.com/ https://upscpdf.com/ https://upscpdf.com/


https://upscpdf.com/ https://upscpdf.com/ https://upscpdf.com/

.
IASBABA’S PRELIMS CURRENT AFFAIRS TEST – MAY 2023

In which of the following above we can Q.92) Consider the following statements
spot snow leopards? about the Sanchar Saathi Portal:

a) One only 1. It was launched by the Ministry of


b) Two only Road Transport and Highways.
c) Three only 2. It enables people across India to
d) Four only track and block their lost or stolen
mobile phones.
Q.90) Consider the following statements
about the National Medical Commission: Choose the correct code:

1. It is a statutory body under the a) 1 only


National Medical Commission Act b) 2 only
of 2019. c) Both 1 and 2
2. The Under-Graduate Medical d) Neither 1 nor 2
Education Board (UGMEB) and the
Post-Graduate Medical Education Q.93) Consider the following:
Board (PGMEB) are under it.
1. Circulating products and materials
Choose the correct code: 2. Regenerating nature
3. Eliminating waste and pollution
a) 1 only
b) 2 only Which of the given above are the principles
c) Both 1 and 2 of circular economy?
d) Neither 1 nor 2
a) 1 and 2
b) 2 and 3
Q.91) Consider the following statements
c) 1 and 3
about the Production Linked Incentive
d) 1, 2 and 3
Scheme:
1. Its objective is to make domestic
Q.94) Consider the following statements
manufacturing globally
about the Trimbakeshwar Mahadev
competitive.
Temple:
2. It is launched only for the large-
scale electronics manufacturing 1. It is located at the source of
sector. the Krishna River.
2. It is the abode of one of the twelve
Choose the correct code:
Jyotirlingas.
a) 1 only 3. It was constructed by the
b) 2 only third Peshwa Balaji Bajirao.
c) Both 1 and 2
Choose the correct code:
d) Neither 1 nor 2
a) 1 and 2

https://upscpdf.com/ https://upscpdf.com/ https://upscpdf.com/


https://upscpdf.com/ https://upscpdf.com/ https://upscpdf.com/

.
IASBABA’S PRELIMS CURRENT AFFAIRS TEST – MAY 2023

b) 2 and 3 2. They have been granted the status


c) 1 and 3 of a Scheduled Tribe.
d) 1, 2 and 3 3. They follow a matrilineal system of
inheritance.
Q.95) Consider the following statements
about Gekko mizoramensis: Choose the correct code:

1. It is arboreal and nocturnal. a) 1 and 2


2. It is found only in India and b) 2 and 3
Bangladesh. c) 1 and 3
d) 1, 2 and 3
Choose the correct code:
Q.98) Consider the following statements
a) 1 only about the Adopt a Heritage: Apni
b) 2 only Dharohar, Apni Pehchaan Scheme:
c) Both 1 and 2
d) Neither 1 nor 2 1. It is an initiative of the Ministry of
Tourism, in collaboration with
Q.96) Consider the following statements the Ministry of Culture.
about London Interbank Offer Rate 2. Private firms, corporates, NGOs,
(LIBOR): and public sector units can adopt
and maintain State-owned
1. It is the global reference rate archaeological sites.
for unsecured long-term
borrowings. Choose the correct code:
2. It is used for the pricing of interest
rate swaps and currency rate a) 1 only
swaps. b) 2 only
c) Both 1 and 2
Choose the correct code: d) Neither 1 nor 2

a) 1 only Q.99) Consider the following statements


b) 2 only about the National Gallery of Modern Art
c) Both 1 and 2 (NGMA):
d) Neither 1 nor 2
1. It aims to acquire and preserve
Q.97) Consider the following statements works of modern art from 1947
about Khasi Tribe: onward.
2. It is run and administered as a
1. They are indigenous matrilineal subordinate office to the Ministry
communities along with Garos and of Culture.
Jaintias.
Choose the correct code:

https://upscpdf.com/ https://upscpdf.com/ https://upscpdf.com/


https://upscpdf.com/ https://upscpdf.com/ https://upscpdf.com/

.
IASBABA’S PRELIMS CURRENT AFFAIRS TEST – MAY 2023

a) 1 only
b) 2 only
c) Both 1 and 2
d) Neither 1 nor 2

Q.100) Consider the following statements


about the Sahyadri Tiger Reserve:

1. It is located in the Sahyadri Ranges


of Western Ghats in Karnataka.
2. It comprises rich evergreen, semi-
evergreen, and moist deciduous
forests.

Choose the correct code:

a) 1 only
b) 2 only
c) Both 1 and 2
d) Neither 1 nor 2

https://upscpdf.com/ https://upscpdf.com/ https://upscpdf.com/


https://upscpdf.com/ https://upscpdf.com/ https://upscpdf.com/

.
IASBABA’S PRELIMS CURRENT AFFAIRS TEST- MAY 2023 SOLUTIONS

Q.1) Solution (b)

Statement Analysis:

Statement 1 Statement 2

Incorrect Correct

No laboratory Centre or clinic will conduct any


The Pre-Conception and Pre-Natal Diagnostic test including ultrasonography for the
Techniques (PCPNDT) Act, 1994 is an act of purpose of determining the sex of the foetus.
the Parliament of India that was enacted No person, including the one who is
to stop female foeticides and arrest the conducting the procedure as per the law,
declining sex ratio in India. will communicate the sex of the foetus to the
The act banned prenatal sex determination. pregnant woman or her relatives by words,
The main objective of enacting the act is signs, or any other method.
to ban the use of sex selection techniques Any person who puts an advertisement for
before or after conception and prevent pre-natal and pre-conception sex
the misuse of prenatal diagnostic techniques determination facilities in the form of a
for sex-selective abortion. notice, circular, label, wrapper, or any
It regulates the use of pre-natal diagnostic document, or advertises through interior or
techniques, like ultrasound machines by other media in electronic or print form or
allowing their use only to detect - genetic engages in any visible representation made by
abnormalities, metabolic disorders, means of hoarding, wall painting, signal, light,
chromosomal abnormalities, certain sound, can be imprisoned for up to three
congenital malformations, years and fined Rs. 10,000.
haemoglobinopathies, and sex-linked
disorders.
Statement 3

Correct

The offences covered under the act include:


 Conducting or aiding in prenatal
diagnostic techniques in unregistered
facilities is an offense under the act.
 Sex selection on a man or woman is
prohibited by the act.
 Performing prenatal diagnostic
techniques for any purpose other than
the one specified in the act is an
offense.

https://upscpdf.com/ https://upscpdf.com/ https://upscpdf.com/


https://upscpdf.com/ https://upscpdf.com/ https://upscpdf.com/

.
IASBABA’S PRELIMS CURRENT AFFAIRS TEST- MAY 2023 SOLUTIONS

 The sale, distribution, supply, renting,


etc. of any ultrasound machine or any
other equipment capable of detecting
the sex of the fetus is prohibited by the
act.
Source: https://www.thehindu.com/news/cities/Delhi/grey-areas-in-law-banning-prenatal-
sex-determination-need-fixing-says-hc/article66796959.ece

Q.2) Solution (d)

Statement Analysis:

Statement 1 Statement 2

Correct Correct

The Indian Institute of Petroleum (IIP), a SAF is created using domestically developed
laboratory of the Council of Scientific and methods using cooking oil and oil-rich seeds
Industrial Research (CSIR), has tied up from plants.
with Boeing, Indigo, Spicejet, and the It can be created using different materials,
three Tata Airlines - Air India, Vistara, such as non-edible and edible oils, as well as
and AirAsia India to support the production used cooking oil.
of Sustainable Aviation Fuel (SAF). They used various sources, including palm
Sustainable Aviation Fuel (SAF), is also stearin, sapium oil, palm fatty acid distillates,
referred to as bio-jet fuel. The SAF algae oil, karanja, and jatropha.
samples produced by the institutes are
undergoing strict testing at the US Federal
Aviation Administration Clearinghouse to
meet the standards required for the ASTM
D4054 certification from ASTM International.
Statement 3

Correct

Benefits of SAF:
 Scaling up the production and use of
SAF in India can bring several benefits,
including reducing GHG emissions,
improving air quality, enhancing
energy security, creating jobs in the
renewable energy sector, and
promoting sustainable development.

https://upscpdf.com/ https://upscpdf.com/ https://upscpdf.com/


https://upscpdf.com/ https://upscpdf.com/ https://upscpdf.com/

.
IASBABA’S PRELIMS CURRENT AFFAIRS TEST- MAY 2023 SOLUTIONS

 It can also help the aviation


industry meet its environmental
targets and contribute to global
efforts to combat climate change.
 It can be mixed with regular jet
fuel and used together.
 Compared to traditional fuel, it
has lower sulfur content, which can
decrease air pollution and support
India's goal of achieving Net Zero
emissions.
Source: https://www.downtoearth.org.in/news/energy/india-s-sustainable-jet-fuel-may-get-
internationally-certified-in-2023-
88994#:~:text=Published%3A%20Thursday%2027%20April%202023&text=A%20bio%2Djet%
20fuel%2C%20also,be%20certified%20internationally%20this%20year.

Q.3) Solution (c)

Statement Analysis:

Statement 1 Statement 2

Correct Correct

Fortifying tea with folate and vitamin B12 may  Fortification is the addition of key
help counter anaemia in Indian women as tea vitamins and minerals such as iron,
is the most common beverage drunk in India. iodine, zinc, and Vitamin A & D to
Vitamin B12 and folate are both important for staple foods such as rice, milk, and salt
the production of red blood cells in the body. to improve their nutritional content.
Vitamin B12 is necessary for the proper These nutrients may or may not have
absorption and utilization of folate in the been originally present in the food
body; folate deficiency can cause severe birth before processing.
defects (NTDs).  Fortification is a safe method of
improving nutrition among people. If
the quantity added is well regulated
as per prescribed standards, the
likelihood of an overdose of nutrients
is unlikely.
 It does not require any changes in food
habits and patterns people and is a
socio-culturally acceptable way to

https://upscpdf.com/ https://upscpdf.com/ https://upscpdf.com/


https://upscpdf.com/ https://upscpdf.com/ https://upscpdf.com/

.
IASBABA’S PRELIMS CURRENT AFFAIRS TEST- MAY 2023 SOLUTIONS

deliver nutrients to people. It also


does not alter the characteristics of
the food—the taste, the feel, the look.
 This method is cost-effective
especially if advantage is taken of the
existing technology and delivery
platforms.
 The Copenhagen Consensus estimates
that every 1 Rupee spent on
fortification results in 9 Rupees in
benefits to the economy.
Source: https://www.thehindu.com/sci-tech/science/anaemia-major-flaws-in-a-tea-
fortification-study/article66790924.ece

Q.4) Solution (c)

Statement Analysis:

Statement 1 Statement 2

Incorrect Incorrect

Solid Waste includes Solid or semi-solid Used sanitary waste like diapers and sanitary
domestic waste, sanitary waste, commercial pads should be wrapped securely in pouches
waste, institutional waste, catering and provided by manufacturers or brand
market waste and other non-residential owners of these products or in a suitable
wastes, street sweepings, silt removed or wrapping material and shall place the same in
collected from the surface drains, the bin meant for dry waste/ non-
horticulture waste, agriculture, and dairy biodegradable waste.
waste treated biomedical waste excluding
industrial waste, bio-medical waste, e-waste,
battery waste, radioactive waste, etc.

The Key Features of Solid Waste Management


Rules, 2016:
 Responsibilities of Generators have
been introduced to segregate waste
into three streams:
 Wet (Biodegradable)
 Dry (Plastic, Paper, metal, wood, etc.)

https://upscpdf.com/ https://upscpdf.com/ https://upscpdf.com/


https://upscpdf.com/ https://upscpdf.com/ https://upscpdf.com/

.
IASBABA’S PRELIMS CURRENT AFFAIRS TEST- MAY 2023 SOLUTIONS

 Domestic hazardous wastes (diapers,


napkins, empty containers of cleaning
agents, mosquito repellents, etc.)
and hand over segregated wastes to
authorized rag-pickers or waste
collectors or local bodies.
 Waste Generators will have to pay:
 ‘User Fee’ to waste collectors.
 ‘Spot Fine’ for Littering and Non-
segregation.
Statement 3 Statement 4

Correct Correct

The bio-degradable waste should All manufacturers of disposable


be processed, treated, and disposed of products such as tin, glass, plastic packaging,
through composting or bio-methanation etc., or brand owners who introduce such
within the premises as far as possible. products in the market shall provide
The residual waste shall be given to the waste necessary financial assistance to local
collectors or agency as directed by the local authorities for the establishment of a waste
authority. management system.

Note:

 Solid Waste Management Rules, 2016 replaced the Municipal Solid Wastes
(Management and Handling) Rules, 2000.
 The Rules are applicable beyond:
 Municipal areas and extend to urban agglomerations,
 Census towns, notified industrial townships,
 Areas under the control of Indian Railways, airports, airbases, ports, and harbour,
 Defence establishments,
 Special economic zones,
 State and Central Government organizations,
 Places of pilgrims, religious and historical importance.
Source: https://www.thehindu.com/sci-tech/energy-and-environment/explained-how-are-
straydog-bites-related-to-poor-waste-management/article66769597.ece

https://upscpdf.com/ https://upscpdf.com/ https://upscpdf.com/


https://upscpdf.com/ https://upscpdf.com/ https://upscpdf.com/

.
IASBABA’S PRELIMS CURRENT AFFAIRS TEST- MAY 2023 SOLUTIONS

Q.5) Solution (b)

Statement Analysis:

Statement 1 Statement 2

Incorrect Correct

The Command Cyber Operations and Support The unit will be responsible for safeguarding
Wings (CCOSWs) are a specialized unit of the the networks and enhancing the
Indian Army. cybersecurity posture of the Indian Army.
It will be instrumental in maintaining They will also facilitate better utilization of
the confidentiality, integrity, and availability modern communication systems and
of critical information. networks within the Indian Army.
Source: https://www.thehindu.com/news/national/army-to-raise-command-cyber-
operations-and-support-wings/article66785107.ece

Q.6) Solution (b)

Statement Analysis:

Statement 1 Statement 2

Incorrect Correct

The Indian government has launched a first- The MEC is a unique concept that will
of-its-kind Millets Experience Centre (MEC) in promote millet as a versatile, healthy grain by
collaboration with the National Agricultural showcasing its dietary benefits and offering
Cooperative Marketing Federation of India customers a unique dining experience.
(NAFED). Visitors to the center can purchase a variety
Millet has been referred to as ‘Shree of ready-to-eat and ready-to-cook
Anna’ in Union Budget 2023-24. products from local millet start-ups.
The MEC will help widen the horizon for
consumers who are actively looking for
healthier alternatives.
Statement 3

Correct

The United Nations General Assembly has


declared 2023 as the International Year of
Millets.
India's proposal to observe an International
Year of Millets in 2023 was approved by

https://upscpdf.com/ https://upscpdf.com/ https://upscpdf.com/


https://upscpdf.com/ https://upscpdf.com/ https://upscpdf.com/

.
IASBABA’S PRELIMS CURRENT AFFAIRS TEST- MAY 2023 SOLUTIONS

the Food and Agriculture Organisation


(FAO) in 2018 and the UNGA has declared the
year 2023 as the International Year of Millets.
This was adopted by a UN ReSolution for
which India took the lead and was supported
by over 70 nations.
Its objective is to create awareness of
the contribution of millet to food security and
nutrition and inspire stakeholders to improve
sustainable production and quality of millet.
Source: https://pib.gov.in/PressReleasePage.aspx?PRID=1920622

Q.7) Solution (a)

Statement Analysis:

Statement 1 Statement 2

Correct Correct

Rabindranath Tagore was a Bengali poet, He believed that nationalism deteriorated


novelist, artist, painter, and educational from patriotism to chauvinism.
theorist. He is credited for giving the Mahatma title to
He is credited with composing the National Mahatma Gandhi.
Anthem of India. He strongly protested against the partition of
Tagore won the Nobel Prize for Literature in Bengal in 1905.
1913 for his collection Gitanjali. He wrote many national songs and attended
He was the first Indian and non-European to protest meetings against the decision to
receive a Nobel Prize. partition Bengal.
He received the British Knighthood in 1915.
However, he renounced it in protest against Tagore advocated the religion of humanity.
the Jallianwalla Bagh massacre in 1919.
He fought against the evils of his society such In his view, traditional schools imprison
as poverty, superstition, and untouchability. children.
He welcomed Western science and Western Hence, he started a model school after the
beliefs in individual worth, freedom, and ancient hermitage schools of India
democracy. named Santiniketan (the abode of peace).
Santiniketan engaged many scholars from
across the world, including his English friends,
Oxford professor E J Thompson, missionary C
F Andrews, and Lord Elmhirst.

https://upscpdf.com/ https://upscpdf.com/ https://upscpdf.com/


https://upscpdf.com/ https://upscpdf.com/ https://upscpdf.com/

.
IASBABA’S PRELIMS CURRENT AFFAIRS TEST- MAY 2023 SOLUTIONS

He also widened his educational commitment


by founding a university – Visva Bharati
Visva Bharati: promoted an international
culture of unity in diversity.
Statement 3

Incorrect

Poems:
Manasi (The Ideal One,1890)
Sonar Tari (The Golden Boat,1894)
Gitanjali (Song Offerings,1910)
Gitimalya (Wreath of Songs,1914)
Balaka (The Flight of Cranes,1916)

Plays:
Raja (1910) [The King of the Dark Chamber],
Dakghar (The Post Office,1912)
Achalayatan (The Immovable,1912)
Muktadhara (The Waterfall,1922)
Raktakaravi (Red Oleanders,1926)

Short stories and Novels:


Gora (1910),
Ghare-Baire (The Home and the World, 1916)
and
Yogayog (Crosscurrents, 1929)
Source: https://www.thehindu.com/news/national/india-bangladesh-share-deep-ties-no-
one-can-undermine-our-bilateral-relations-says-amit-shah/article66830086.ece

Q. 8) Solution (a)

Statement Analysis:

Statement 1 Statement 2

Correct Incorrect

The Geological Survey of India (GSI) is a It works under the Ministry of Mines.
scientific agency of India founded in 1851. Headquarters: Kolkata
Background: It was set up in 1851 primarily to
find coal deposits for the Railways.

https://upscpdf.com/ https://upscpdf.com/ https://upscpdf.com/


https://upscpdf.com/ https://upscpdf.com/ https://upscpdf.com/

.
IASBABA’S PRELIMS CURRENT AFFAIRS TEST- MAY 2023 SOLUTIONS

It is one of the oldest organisations in the It has six regional offices located in Lucknow,
world and the second oldest survey in Jaipur, Nagpur, Hyderabad, Shillong, and
India after the Survey of India. Kolkata.
Objectives:-
 Conducting geological surveys and
studies of India.
 Being the prime provider of basic
earth science information to
government, industry, and the general
public, as well as the official
participant in steel, coal, metals,
cement, power industries, and
international geoscientific forums.
Functions: creation and updation of national
geoscientific information and mineral
resource assessment.
Source: https://economictimes.indiatimes.com/industry/indl-goods/svs/metals-mining/gsi-
terms-reports-on-large-lithium-reserves-found-in-rajasthan-as-
baseless/articleshow/100109469.cms

Q.9) Solution (d)

Statement Analysis:

Statement 1 Statement 2

Correct Correct

The Economic and Social Commission for Asia ESCAP is one of the five regional commissions
and the Pacific (ESCAP) is the most of the United Nations.
inclusive intergovernmental platform in the The other four regional groups of the United
Asia-Pacific region. Nations are the African Group, Eastern
The United Nations Economic and Social European Group, Latin American and
Commission for Asia and the Pacific is Caribbean Group, and Western European.
the main legislative organ of ESCAP.
The Commission promotes cooperation in
pursuit of Solutions to sustainable
development challenges.
It provides a forum for all governments of the
region to review and discuss economic and

https://upscpdf.com/ https://upscpdf.com/ https://upscpdf.com/


https://upscpdf.com/ https://upscpdf.com/ https://upscpdf.com/

.
IASBABA’S PRELIMS CURRENT AFFAIRS TEST- MAY 2023 SOLUTIONS

social issues and to strengthen regional


cooperation.
The Commission meets annually at the
ministerial level.
It discusses and decides on important issues
pertaining to inclusive and sustainable
economic and social development in the
region, to decide on the recommendations of
its subsidiary bodies and the Executive
Secretary.
It reviews and endorses the proposed
strategic framework and programme of work,
and makes any other decisions required, in
conformity with its terms of reference.
Statement 3

Incorrect

It reports to the Economic and Social Council


(ECOSOC) of the UN.
It has 53 member States and 9 associate
members.
Functions:-
 It supports inclusive, resilient, and
sustainable development in the region
by generating action-oriented
knowledge.
 It provides technical assistance and
capacity-building services in support
of national development objectives,
regional agreements, and
the implementation of the 2030
Agenda for Sustainable Development.
 It also provides support to partners at
the national level.
Source: https://www.downtoearth.org.in/news/natural-disasters/most-asia-pacific-
countries-ill-prepared-for-natural-disasters-escap-89186

https://upscpdf.com/ https://upscpdf.com/ https://upscpdf.com/


https://upscpdf.com/ https://upscpdf.com/ https://upscpdf.com/

.
IASBABA’S PRELIMS CURRENT AFFAIRS TEST- MAY 2023 SOLUTIONS

Q.10) Solution (a)

Statement Analysis:

Statement 1 Statement 2

Correct Incorrect

The Battle of Haldighati was fought in 1576. Maharana Pratap fought a brave war but was
It was between Rana Pratap Singh of Mewar defeated by Mughal forces.
and Raja Man Singh of Amber who was the It is said that Maharana Pratap’s loyal horse
general of the Mughal emperor Akbar. named Chetak, gave up his life as the
Maharana was leaving the battlefield.

Reconquest:-
 After 1579, Pratap recovered Western
Mewar including Kumbhalgarh,
Udaipur, and Gogunda.
 He also built a new capital, Chavand,
near modern Dungarpur.
 He was succeeded by his son Amar
Singh, who submitted in 1614
to Emperor Jahangir.

Note:
 Maharana Pratap was born in Kumbhalgarh, Rajasthan.
 He was the 13th King of Mewar and the eldest son of Udai Singh II.
 Maharana Udai Singh II: ruled the kingdom of Mewar, with his capital at Chittor.
 Udai Singh II was also a founder of the city of Udaipur (Rajasthan).
Source: https://economictimes.indiatimes.com/news/new-updates/maharana-pratap-
jayanti-2023-heres-everything-you-need-to-know-about-the-
day/articleshow/100093201.cms

Q.11) Solution (b)

Statement Analysis:

Statement 1 Statement 2

Incorrect Correct

https://upscpdf.com/ https://upscpdf.com/ https://upscpdf.com/


https://upscpdf.com/ https://upscpdf.com/ https://upscpdf.com/

.
IASBABA’S PRELIMS CURRENT AFFAIRS TEST- MAY 2023 SOLUTIONS

Coco Islands are a small group of islands Coco Islands are geologically an extended
located in the Bay of Bengal. division of the Arakan Mountains or Rakhine
They are part of the Yangon Region of Mountains.
Myanmar. They submerge as a chain of islands in the Bay
of Bengal for a long stretch and emerge again
in the form of the Andaman and Nicobar
Islands.
They are part of the same topography as
India’s Andaman & Nicobar Islands.
Great Coco Island, the largest in the group,
lies just 55 km from India’s strategic Andaman
and Nicobar Islands.
Statement 3

Correct

In the early 19th century, the British


government in India established a penal
colony in the Andaman for the convicts in the
Indian subcontinent, and the Coco Islands
were a source of food for it.
The British government had reportedly leased
out the islands to the Jadwet family of Burma.
The leasing of control of the Coco Islands
resulted in poor governance of the islands,
which made the British government in
India transfer its control to the government of
Lower Burma in Rangoon.
In 1882, the islands officially became part of
British Burma.
The islands became a self-governing crown
colony even after Burma was separated from
British India in 1937.
Source: https://www.indiatoday.in/india/story/indian-navy-drone-keeps-eye-on-china-
linked-coco-islands-after-top-myanmar-military-officials-visit-2376840-2023-05-09

https://upscpdf.com/ https://upscpdf.com/ https://upscpdf.com/


https://upscpdf.com/ https://upscpdf.com/ https://upscpdf.com/

.
IASBABA’S PRELIMS CURRENT AFFAIRS TEST- MAY 2023 SOLUTIONS

Q.12) Solution (d)

Statement Analysis:

Statement 1 Statement 2

Correct Correct

The Gem & Jewellery Export Promotion Its objective is to promote the Indian brand by
Council is the apex body founded in 1966. organising India’s biggest and second-biggest
It drives India’s export-led growth in the gem trade shows, IIJS Premiere and IIJS Signature.
and jewellery sector. It facilitates better interaction on trade-
It is headquartered in Mumbai. related issues between the industry and the
It has regional offices across the country and Government of India, Ministry of Commerce
over 7,500 members in its fold. & Industry, Ministry of Finance, DGFT, Dept of
Its vision is to make India the preferred source Commerce, and Dept. of Finance.
of quality gems and jewellery.
Statement 3

Correct

It upholds diamond integrity by executing


the Kimberly Process Certification Scheme in
the country, as the nodal agency appointed by
the Government of India.
Source: https://newsonair.gov.in/News?title=GJEPC-launches-India-Jewellery-Exposition-
2023-in-Dubai-to-promote-%22Made-in-India%22-gems-and-jewellery&id=460657

Q.13) Solution (a)

India jumps 6 places on World Bank Logistic Performance Index.

About Logistic Performance Index: -

 The Logistics Performance Index (LPI), developed by the World Bank Group

 It is the weighted average of the country’s scores on the six key dimensions:

1. Efficiency of the clearance process (i.e., speed, simplicity, and predictability of


formalities) by border control agencies, including customs.
2. Quality of trade and transport-related infrastructure (e.g., ports, railroads,
roads, information technology).
3. Ease of arranging competitively priced shipments.

https://upscpdf.com/ https://upscpdf.com/ https://upscpdf.com/


https://upscpdf.com/ https://upscpdf.com/ https://upscpdf.com/

.
IASBABA’S PRELIMS CURRENT AFFAIRS TEST- MAY 2023 SOLUTIONS

4. Competence and quality of logistics services (e.g., transport operators,


customs brokers).
5. Ability to track and trace consignments.
6. Timeliness of shipments in reaching destinations within the scheduled or
expected delivery time.

 India improved in 4 out of 6 indicators.

 The LPI 2023 allows for comparisons across 139 countries.


 The 2023 LPI for the first time measures the speed of trade with indicators derived
from big datasets tracking shipments.
 India’s new ranking on Logistics Performance Index 2023 is 38. The ranking list was
topped by Singapore.
 India was ranked 44th on the index in 2018 and has now climbed to 38th in the 2023
listing.
 According to the report, India’s rank moved up five places in infrastructure score from
52nd in 2018 to 47th in 2023.
 It climbed to the 22nd spot for international shipments in 2023 from 44th in 2018 and
moved four places up to 48th in logistics competence and equality.
 In 2022, the prime minister launched the National Logistics Policy (NLP) to ensure
quick last-mile delivery, end transport-related challenges, save time and money for
the manufacturing sector and ensure desired speed in the logistics sector.
 According to the report, the average dwell time for containers between May and
October 2022 was three days for India and Singapore, much better than some of the
industrialised countries. The dwell time for the US was seven days and for Germany,
it was 10 days.

https://indianexpress.com/article/india/india-world-bank-logistic-performance-
index-8571441/

Q.14) Solution (d)

Statement Analysis:

Statement 1 Statement 2

Correct Correct

Saint Samarth Ramdas was an Indian Marathi His literary works include Karunashtakas,
Hindu saint, poet, philosopher, writer, and Dasbodh, Yuddhakand,
spiritual master. Sunderkand, Poorvarambh, Antarbhav,
Chaturthman, Aatmaaram, Panchman,

https://upscpdf.com/ https://upscpdf.com/ https://upscpdf.com/


https://upscpdf.com/ https://upscpdf.com/ https://upscpdf.com/

.
IASBABA’S PRELIMS CURRENT AFFAIRS TEST- MAY 2023 SOLUTIONS

He was a devotee of Hindu deities Rama and Panchsamasi, Manpanchak,


Hanuman. Janaswabhawgosavi, etc.
He was previously famous as Narayan.
He was born in Jamb, a village in the present-
day Jalna district of Maharashtra.
He was born on the occasion of Rama Navami,
in 1608.
Ramdas served as an inspiration for several
Indian thinkers, historians, and social
reformers from the 19th and 20th-century,
including Bal Gangadhar Tilak, Rajwade,
Keshav Hedgewar, and Ramchandra Ranade.
A spiritual guru, Nana Dharmadhikari,
promoted the views of Ramdas through his
spiritual discourses.
Statement 3

Correct

Ramdas is not deemed a pacifist.


His writings include strong expressions
encouraging nationalism to counter
aggressive Muslim invaders.
Source: https://indianexpress.com/article/explained/explained-history/samarth-ramdas-
chhatrapati-shivaji-maharaj-8598333/

Q.15) Solution (b)

Statement Analysis:

Statement 1 Statement 2

Incorrect Correct

Gold Rush is the phenomenon of adding gold Its objective is to safeguard returns on gold
to the central bank’s reserves at an unusual amid global uncertainty and a rising inflation
pace. scenario.
Adding gold to its reserves is considered
a more safe, more secure, liquid asset.
Reserve Bank of India’s (RBI) gold reserves
showed an increase of nearly 5 percent over

https://upscpdf.com/ https://upscpdf.com/ https://upscpdf.com/


https://upscpdf.com/ https://upscpdf.com/ https://upscpdf.com/

.
IASBABA’S PRELIMS CURRENT AFFAIRS TEST- MAY 2023 SOLUTIONS

fiscal 2022 when it held 760.42 metric tonnes


of gold.

Source: https://indianexpress.com/article/explained/explained-economics/rbis-hoard-of-
gold-now-almost-800-tonnes-whats-behind-the-gold-rush-by-central-banks-8599547/

Q.16) Solution (b)

Statement Analysis:

Statement 1 Statement 2

Incorrect Correct

Palak Lake is the largest natural lake in The Palak Lake wetland conservation area
Mizoram. falls under the Indo-Burma biodiversity
It is situated near Phura village which is about
hotspot.
391 km to the south of Aizawl. It is rich in biodiversity.
It is oval in shape and covers about 1 square It is surrounded by lush virgin forests rich in
km. flora and fauna.
It is home to most of the common wetland
birds and hill birds
It is believed to be a winter stop-over for
migrating Pintail Duck.
A few elephants still roam the surrounding
virgin forests.
Source: https://pib.gov.in/PressReleasePage.aspx?PRID=1923504

Q.17) Solution (b)

Statement Analysis:

Statement 1 Statement 2

Incorrect Correct

Fomalhaut is located 25 light-years from Earth It lies in the southern constellation Piscis
It was one of the first exoplanets ever Austrinus, 25 light-years from Earth.
discovered in visible light by NASA’s Hubble It is the brightest star in the southern
Space Telescope in 2004. constellation Piscis Austrinus.
Exoplanets are planets that orbit around stars
other than the Sun.

https://upscpdf.com/ https://upscpdf.com/ https://upscpdf.com/


https://upscpdf.com/ https://upscpdf.com/ https://upscpdf.com/

.
IASBABA’S PRELIMS CURRENT AFFAIRS TEST- MAY 2023 SOLUTIONS

It is surrounded by debris disks that are


remnants of collisions between larger bodies,
similar to asteroids and comets.
Astronomers proposed that Fomalhaut b was
not a real planet but rather an expanding dust
cloud left behind by a collision between two
cometlike bodies.
Statement 3

Correct

It is used in navigation because of its


conspicuous place in a sky region otherwise
lacking in bright stars.
It is a white star, it has an apparent magnitude
of 1.16.
Source: https://www.timesnownews.com/technology-science/james-webb-space-
telescope-discovers-amazing-dust-rings-around-distant-star-article-100096528

Q.18) Solution (d)

Statement Analysis:

Statement 1 Statement 2

Correct Correct

Central Board of Indirect Taxes and Customs It is a statutory body established under
is a part of the Department of Revenue under the Central Boards of Revenue Act, of 1963.
the Ministry of Finance. It was formed in 1964 when the Central Board
Functions performed by the CBIC include:- of Revenue was split into the Central Board of
Formulation of policy concerning levy and Direct Taxes (CBDT) and the Central Board of
collection of Customs, Central Excise duties, Excise and Customs.
Central Goods & Services Tax, and IGST. The Central Board of Excise and Customs was
Prevention of smuggling. renamed the Central Board of Indirect Taxes
Administration of matters relating and Customs in 2018.
to Customs, Central Excise, Central Goods &
Services Tax, IGST, and Narcotics to the extent
under CBIC’s purview.
Statement 3

Correct

https://upscpdf.com/ https://upscpdf.com/ https://upscpdf.com/


https://upscpdf.com/ https://upscpdf.com/ https://upscpdf.com/

.
IASBABA’S PRELIMS CURRENT AFFAIRS TEST- MAY 2023 SOLUTIONS

It is the nodal national agency responsible for


administering:-
 Customs
 GST
 Central Excise
 Service Tax
 Narcotics in India.
The Board is the administrative authority for
its subordinate organizations, including
Custom Houses, Central Excise and Central
GST Commissionerate, and the Central
Revenues Control Laboratory.
Source: https://www.business-standard.com/economy/news/cbic-rolls-out-module-for-
automated-scrutiny-of-gst-returns-of-taxpayers-123051101005_1.html

Q.19) Solution (a)

Statement Analysis:

Statement 1 Statement 2

Incorrect Correct

The Directorate General of Civil Aviation DGCA is responsible for the regulation of air
(DGCA) is a statutory body of the Government transport services to/from/within India and
of India. for the enforcement of civil air regulations, air
It was formed under the Aircraft safety, and airworthiness standards.
(Amendment) Act, of 2020. Functions:-
It comes under the Ministry of Civil Aviation.  Registration of civil aircraft.
 Formulation of standards of
airworthiness for civil aircraft
registered in India and grant of
certificates of airworthiness to such
aircraft.
 Licensing of pilots, aircraft
maintenance engineers, and flight
engineers, and conducting
examinations and checks for that
purpose.
 Licensing of air traffic controllers.
 Investigating accidents/incidents.

https://upscpdf.com/ https://upscpdf.com/ https://upscpdf.com/


https://upscpdf.com/ https://upscpdf.com/ https://upscpdf.com/

.
IASBABA’S PRELIMS CURRENT AFFAIRS TEST- MAY 2023 SOLUTIONS

 Taking accident prevention measures.


 Coordination at the national level for
flexible use of air space by civil and
military air traffic agencies
 Interaction with ICAO for the provision
of more air routes for civil use through
Indian air space.
 Promoting indigenous design and
manufacture of aircraft.
Statement 3

Correct

It aims to regulate civil aviation in India.


It primarily deals with safety issues in civil
aviation.
Its headquarters are located in New Delhi.
It has regional offices in various parts of India.
Source: https://www.hindustantimes.com/india-news/air-india-pilot-s-license-suspended-
for-cockpit-breach-airline-fined-30-lakh-for-safety-violation-dgca-issues-warning-
101683916724581.html

Q.20) Solution (d)


Statement Analysis:

The World Health Organization (WHO) has launched a new initiative to be better prepared
for future outbreaks of a similar scale and devastation as the COVID-19 pandemic.
The Preparedness and Resilience for Emerging Threats (PRET) Initiative is aimed at
providing “guidance on integrated planning for responding to any respiratory pathogen such
as influenza or coronaviruses.”

The current focus of PRET will be on respiratory viruses — in the backdrop of the SARS-
CoV-2 outbreak.

The initiative was announced at the Global Meeting for Future Respiratory Pathogen
Pandemics held on 24-26 April 2023 in Geneva, Switzerland.

PRET is an evolution in WHO’s approach to pandemic preparedness through the application


of a mode of transmission lens, rather than a focus on specific diseases.
About WHO It was established on April 7, 1948, with HQ in Geneva, Switzerland.
https://www.who.int/news/item/26-04-2023-who-launches-new-initiative-to-improve-
pandemic-preparedness

https://upscpdf.com/ https://upscpdf.com/ https://upscpdf.com/


https://upscpdf.com/ https://upscpdf.com/ https://upscpdf.com/

.
IASBABA’S PRELIMS CURRENT AFFAIRS TEST- MAY 2023 SOLUTIONS

Q.21) Solution (b)

Statement Analysis:

Statement 1 and 2 Statement 3

Incorrect Correct

The Visva-Bharati University was set up Rabindranath Tagore believed in open-air


by Nobel laureate Rabindranath Tagore in education.
1921 at Santiniketan, West Bengal. He had reservations about any teaching done
He called it Visva-Bharati, which means within four walls.
the communion of the world with India. This was due to his belief that walls represent
Until independence, it was a college. the conditioning of the mind.
Soon after independence, the institution was Tagore did not have a good opinion about the
given the status of a central university in Western method of education introduced by
1951 by an act of Parliament. the British in India.
On this subject, Tagore and Gandhiji’s
opinions matched.
So, he devised a new system of learning in
Visva-Bharati.
The special feature of Visva Bharti:-
 He allowed students to continue their
course till the student and his teacher
both are satisfied.
 At Visva-Bharati, if a course demanded
by a student is not available, then the
university will design a course and
bring teachers for that course.
 The university would not be bothered
by the consideration of whether there
is a demand for the course.
Source: https://www.indiatoday.in/education-today/news/story/visva-bharati-set-to-
become-first-living-heritage-university-to-receive-unesco-world-heritage-tag-2378379-
2023-05-12

https://upscpdf.com/ https://upscpdf.com/ https://upscpdf.com/


https://upscpdf.com/ https://upscpdf.com/ https://upscpdf.com/

.
IASBABA’S PRELIMS CURRENT AFFAIRS TEST- MAY 2023 SOLUTIONS

Q.22) Solution (a)

Statement Analysis:

Statement 1 Statement 2

Correct Incorrect

Monlam Chenmo is an annual five-day-long


It is a mass prayer camp by Buddhist monks
Great Prayer Festival of Ladakh. and nuns for world peace and happiness.
The Festival falls on the 21st to 25th days of
The annual Mass Prayer is resuming this year
the third month of the Tibetan lunar calendar.
after three years of discontinuation due to
It is being held since 1991. Covid.
The event is being organised by the All Ladakh
Gonpa Association.
It is a major annual prayer of the Sangha
Community of Ladakh gathered at one
platform.
Source: https://newsonair.gov.in/News?title=Ladakh-annual-festival-
%26%2339%3bMonlam-Chenmo%26%2339%3b-begins-in-Leh&id=460746

Q.23) Solution (c)

Statement Analysis:

Statement 1 Statement 2

Correct Incorrect

Hydrogen sulphide (H2S) is a colourless It dissolves in water and oil.


chalcogen hydride gas with a It may be released when these liquids are
characteristic foul odour. heated, depressurized, or agitated.
It is very poisonous, corrosive, and It occurs in volcanic gases, natural gas,
flammable. and some sources of well water.
Statement 3

Correct

Hydrogen sulphide is used primarily to


produce sulfuric acid and sulphur.
It is also used to create a variety of inorganic
sulphides.
It is used to create pesticides, leather, dyes,
and pharmaceuticals.

https://upscpdf.com/ https://upscpdf.com/ https://upscpdf.com/


https://upscpdf.com/ https://upscpdf.com/ https://upscpdf.com/

.
IASBABA’S PRELIMS CURRENT AFFAIRS TEST- MAY 2023 SOLUTIONS

It is used to produce heavy water for nuclear


power plants.
It is also used or is a by-product in
many industrial processes.
Hydrogen sulfide is produced naturally from
decaying organic matter.
It occurs naturally in sewers, manure pits,
well water, oil and gas wells, and volcanoes.
It can be released from sewage sludge, liquid
manure, sulfur hot springs, and natural gas.
Source: https://www.downtoearth.org.in/news/governance/foul-smell-to-neighbours-
dropping-dead-what-happened-in-the-15-minutes-giaspura-residents-had-to-escape-a-
deadly-gas-89242

Q.24) Solution (b)

Statement Analysis:

Statement 1 Statement 2

Incorrect Correct

Poshan Bhi, Padhai Bhi is an initiative Its objective is to ensure that India has the
for strengthening Early Childhood Care and world’s largest, universal, high-quality
Education (ECCE). preschool network, as suggested by the NEP
It is launched by the Ministry of Women and 2020.
Child Development. It emphasizes the links with primary
education as well as early childhood health
and nutrition services.
Source: https://indianexpress.com/article/india/poshan-bhi-padhai-bhi-anganwadi-scheme-
for-early-childhood-care-education-launched-8602912/

Q.25) Solution (a)

Statement Analysis:

Statement 1 Statement 2

Correct Incorrect

https://upscpdf.com/ https://upscpdf.com/ https://upscpdf.com/


https://upscpdf.com/ https://upscpdf.com/ https://upscpdf.com/

.
IASBABA’S PRELIMS CURRENT AFFAIRS TEST- MAY 2023 SOLUTIONS

The iDrone initiative aims to transport The iDrone initiative is an innovative of


essential medical resources to remote areas the Indian Council of Medical Research
within the country. (ICMR).
The medical supplies delivered under the The ICMR introduced the i-DRONE during
iDrone project included COVID-19 vaccines, the COVID-19 pandemic.
vaccines used in routine immunisation It is now being used to deliver blood and
programs, antenatal care medicines, blood-related products which are supposed
multivitamins, syringes, and gloves. to be kept at a low temperature.
The drone delivery system focused on an end- The inaugural trial flight carried ten units of
to-end ecosystem for drone-based logistic whole blood samples from the Government
transportation within the states. Institute of Medical Sciences (GIMS) and Lady
It was the first successful example of Hardinge Medical College (LHMC) in a visual
delivering vaccines through drones from land line of sight.
to islands in South Asia.
The longest drone flight under this project
carried 3525 units of medical supplies
from Mokokchung to the district Tuensang in
Nagaland (approx. 40 km).
It will revolutionize the healthcare industry,
especially in remote areas where access to
medical facilities is limited.
Source: https://www.thehindu.com/news/national/icmr-conducts-successful-trial-run-of-
blood-bag-delivery-under-idrone-initiative/article66835394.ece

Q.26) Solution (d)

Statement Analysis:

 The Gajapati irrigation project is a multipurpose irrigation project at Chheligada in the


Gajapati district of Odisha.
 It is being undertaken across the river Badjhore.
 Badjhore: is a tributary of river Vansadhara near the village Chheligada in the Gajapati
District of Odisha.
 The Project envisages the construction of a 250m long & 30m high dam across the
river Badjhore with a central spillway.
 After this Project, 5201 hectares metres of water can be preserved and water can be
supplied for irrigation of 5760 hectares of land in Ganjam and 500 hectares of land in
Gajapati districts.
 This Project will also provide drinking water to Brahampur City.
 Additionally, 36 MW of electricity can be produced through a mini hydel project in
three places Shiali Loti, Kankata, and Dekili in the Gajapati district.

https://upscpdf.com/ https://upscpdf.com/ https://upscpdf.com/


https://upscpdf.com/ https://upscpdf.com/ https://upscpdf.com/

.
IASBABA’S PRELIMS CURRENT AFFAIRS TEST- MAY 2023 SOLUTIONS

 Salient features of the Project are:-


 90m long centrally located Ogee-type gated Spillway
 1.13 km long tunnel to connect Chheligada reservoir with Ghodahada river.
 Canal system directly from the dam to irrigate the Gajapati district.
 Laying of the pipeline for drinking water supply to Berhampur (Distt. Ganjam)
Source: https://timesofindia.indiatimes.com/city/bhubaneswar/move-to-start-rs-936-crore-
gajapati-irrigation-project-in-odisha/articleshow/100146256.cms?from=mdr

Q.27) Solution (c)

Statement Analysis:

Statement 1 Statement 2

Correct Correct

The Central Electricity Authority (CEA) is a It advises the government on matters relating
statutory organization. to the National Electricity Policy (NEP).
It was constituted under Section 3 (1) of the It formulates short-term and prospective
repealed Electricity (Supply) Act, of 1948, and plans for the development of electrical
continued under Section 70 of the Electricity systems.
Act, of 2003. It is the designated authority for cross border
It was established as a part-time body in 1951. trade of electricity.
It became a full-time body in the year 1975. It also prescribes the standards on matters
Members:- such as the construction of electrical plants,
As per Section 70 (3) of the Electricity Act, electric lines and connectivity to the grid,
2003, the authority shall consist of not more safety, and grid standards, and installation
than 14 members and operation of meters.
This includes its chairperson. It is also responsible for the concurrence of
Not more than eight of them shall be full-time hydropower development schemes of
members to be appointed by the Central central, state, and private sectors for the
Government. efficient development of rivers and their
The CEA is headed by a chairperson who, is tributaries for power generation.
the Chief Executive of the authority.
He oversees largely the development of the
power sector in the country.
Source: https://economictimes.indiatimes.com/industry/energy/power/central-electricity-
authority-revises-down-fy30-peak-power-demand-projection-a-
tad/articleshow/99995871.cms?from=mdr

https://upscpdf.com/ https://upscpdf.com/ https://upscpdf.com/


https://upscpdf.com/ https://upscpdf.com/ https://upscpdf.com/

.
IASBABA’S PRELIMS CURRENT AFFAIRS TEST- MAY 2023 SOLUTIONS

Q.28) Solution (b)

Statement Analysis:

Statement 1 Statement 2

Incorrect Correct

The Galápagos Islands are a chain of islands in They are part of the country of Ecuador, in
the Pacific Ocean. South America.
Repeated volcanic eruptions helped to form There are thirteen major islands and a
the rugged mountain landscape of the handful of smaller islands that make up the
Galápagos Islands. Galápagos archipelago.
The Galápagos Islands are located near the The largest of the islands is called Isabela.
equator, yet they receive cool ocean
currents.
This makes for a strange mix of tropical and
temperate climates.
British naturalist Charles Darwin came to the
Galápagos in 1835, on a ship called the HMS
Beagle.
His observations of wildlife on the island
inspired his theory of evolution by natural
selection.
Source: https://indianexpress.com/article/explained/explained-climate/what-is-the-credit-
suisse-ecuador-deal-for-galapagos-conservation-8595418/

Q.29) Solution (b)

Statement Analysis:

Statement 1 Statement 2

Incorrect Correct

The Arab League, formally known as The founding member states were Egypt,
the League of Arab States, was established Syria, Lebanon, Iraq, Transjordan (now
in 1945. Jordan), Saudi Arabia, and Yemen.
It is a regional organization of Arab states in Currently, it has 22 member states, who have
the Middle East and parts of Africa. pledged to cooperate on economic and
military affairs, among other issues.
Other members are Libya (1953); Sudan
(1956); Tunisia and Morocco (1958); Kuwait

https://upscpdf.com/ https://upscpdf.com/ https://upscpdf.com/


https://upscpdf.com/ https://upscpdf.com/ https://upscpdf.com/

.
IASBABA’S PRELIMS CURRENT AFFAIRS TEST- MAY 2023 SOLUTIONS

(1961); Algeria (1962); Bahrain, Oman, Qatar,


and the United Arab Emirates (1971);
Mauritania (1973); Somalia (1974); the
Palestine Liberation Organization (PLO; 1976);
Djibouti (1977); and Comoros (1993).
Each member has one vote on the League
Council, decisions being binding only on those
states that have voted for them.
The signing on April 13, 1950, of
an agreement on joint defence and economic
cooperation also committed the signatories
to the coordination of military defences
measures.
Source: https://indianexpress.com/article/explained/explained-global/arab-league-
reinstates-syria-explained-8598338/

Q.30) Solution (b)

Statement Analysis:

Statement 1 Statement 2

Incorrect Correct

Scary Barbie is a supermassive black hole that The researchers discovered it using an AI
is devouring a star with a thousand times the engine called REFITT (Recommender Engine
brightness of a supernova. For Intelligent Transient Tracking).
It was first observed in 2020. The AI engine looks through observations
It was named ZTF20abrbeie. from many different telescopes around the
It is one of the most luminous, energetic, World.
long-lasting transient objects in the sky.
It is also extremely far away and in a
somewhat neglected corner of the sky.
Source: https://indianexpress.com/article/technology/science/scary-barbie-supermassive-
black-hole-8582245/

Q.31) Solution (d)

Statement Analysis:

 Donanemab is a drug developed for Alzheimer’s by a US pharma company ‘Eli Lilly’.


 It is yet to be approved in the USA.

https://upscpdf.com/ https://upscpdf.com/ https://upscpdf.com/


https://upscpdf.com/ https://upscpdf.com/ https://upscpdf.com/

.
IASBABA’S PRELIMS CURRENT AFFAIRS TEST- MAY 2023 SOLUTIONS

 Donanemab is not a cure for Alzheimer’s.


 The drug aims to remove the plaques from the brain and slow the progression of the
disease.
 It slows cognitive decline by 35%
 Mechanism: It has antibodies that target different forms of amyloid-beta (Aβ)
proteins that can clump together to form amyloid plaques in people’s brains, resulting
in their cognitive decline.
 It has similar antibody-based therapy like lecanemab, but targets different forms of
amyloid-beta (Aβ) proteins.
 Lecanemab:-
 It is a drug developed for Alzheimer’s.
 It is approved in the US.
 It is still under evaluation by the FDA’s counterpart in Europe, the European Medicines
Agency (EMA).
 Lecanemab trial results showed it slowed cognitive decline by 27%.
 Both donanemab and lecanemab are given intravenously.
 Both donanemab and lecanemab have a high risk of side effects like ‘amyloid-related
imaging abnormalities’ (ARIA) which include swelling or bleeding in the brain.

Note: Alzheimer’s disease is a neurological disorder which causes brain cells to degenerate
and die. This leads to loss of memory, problems with words in speaking or writing, poor
judgment, changes in mood and personality, confusion with time or place, etc. It is a brain
disorder that gets worse over time. It is the most common cause of dementia among older
adults.
Source: https://indianexpress.com/article/explained/explained-health/new-alzheimer-
drugs-donanemab-lecanemab-comparison-8596341/

Q.32) Solution (a)

Statement Analysis:

Statement 1 Statement 2

Correct Incorrect

START is an introductory-level training Space Science and Technology Awareness


program for students. Training (START), is launched by the Indian
Objective:- Space Research Organisation (ISRO).
 To provide students
with introductory-level training in
Space Science and Technology.

https://upscpdf.com/ https://upscpdf.com/ https://upscpdf.com/


https://upscpdf.com/ https://upscpdf.com/ https://upscpdf.com/

.
IASBABA’S PRELIMS CURRENT AFFAIRS TEST- MAY 2023 SOLUTIONS

 Giving students an overview of


different facets of the field, research
opportunities, and career options.
The programme will cover various domains of
space science, including Astronomy &
Astrophysics, Heliophysics & Sun-Earth
interaction, Instrumentation, and Aeronomy.
It will be delivered by scientists from Indian
academia and ISRO centres.
Source: https://www.thehindu.com/news/national/karnataka/isro-to-start-online-training-
programme-for-pg-and-and-final-year-ug-students/article66823617.ece

Q.33) Solution (b)

Statement Analysis:

Statement 1 Statement 2

Incorrect Correct

Thalassaemia is a genetic blood Thalassemia can cause anaemia, leading to


disorder affecting the production of red blood fatigue.
cells. Other symptoms:
Genetic Disorder is a disorder which is A pale appearance or yellow colour to the skin
inherited and can be passed down from one (jaundice)
generation to the next. Irritability
It is a genetic blood disorder that causes the Deformities of the facial bones
body to have less haemoglobin than normal. Slow growth
Hemoglobin enables red blood cells to carry A swollen abdomen
oxygen. Dark urine
Abnormal production of blood means that
affected individuals do not make adequate
amounts of functional red blood cells.
Statement 3

Correct

It can be treated through:


 Bone marrow transplant (BMT)
In BMT, high-dose chemotherapy
eliminates thalassemia-producing
cells in the marrow and replaces them

https://upscpdf.com/ https://upscpdf.com/ https://upscpdf.com/


https://upscpdf.com/ https://upscpdf.com/ https://upscpdf.com/

.
IASBABA’S PRELIMS CURRENT AFFAIRS TEST- MAY 2023 SOLUTIONS

with healthy donor cells from bone


marrow or umbilical cord blood.
 Supplements and Medications
 Blood transfusions
Source: https://timesofindia.indiatimes.com/city/nagpur/digital-social-efforts-to-stop-
thalassaemia-gaining-ground/articleshow/100061091.cms

Q.34) Solution (a)

Statement Analysis:

Statement 1 Statement 2

Correct Incorrect

ICI measures the combined and individual These Eight Core Industries comprise 40.27
performance of the production of eight core percent of the weight of items included in the
industries. Index of Industrial Production (IIP).
These include Coal, Crude Oil, Natural Gas,
Refinery Products, Fertilizers, Steel, Cement,
and Electricity.
It is compiled and released by the Office of
Economic Adviser (OEA), Department for
Promotion of Industry and Internal Trade
(DPIIT), Ministry of Commerce and Industry.
It provides an advanced indication of the
production performance of industries of a
‘core’ nature before the IIP.
Source:
https://pib.gov.in/PressReleaseIframePage.aspx?PRID=1920543#:~:text=ICI%20measures%2
0combined%20and%20individual,of%20Industrial%20Production%20(IIP)

Q.35) Solution (c)

Statement Analysis:

Statement 1 Statement 2

Correct Incorrect

https://upscpdf.com/ https://upscpdf.com/ https://upscpdf.com/


https://upscpdf.com/ https://upscpdf.com/ https://upscpdf.com/

.
IASBABA’S PRELIMS CURRENT AFFAIRS TEST- MAY 2023 SOLUTIONS

The International Civil Aviation Organization Its headquarters is located in the Quartier
(ICAO) is a specialized funding agency of the International of Montreal, Quebec, Canada.
United Nations. The Air Navigation Commission (ANC) is the
It changes the principles and techniques of technical body within ICAO.
international air navigation. ICAO defines the protocols for air accident
It fosters the planning and development of investigation that are followed by transport
international air transport to ensure safe and safety authorities in countries signatory to
orderly growth. the Chicago Convention on International Civil
Aviation.
Statement 3

Correct

It has 193 Member States.


India is one of ICAO’s founder members,
having attended the Chicago Convention in
1944.
The ICAO Council adopts standards and
recommended practices concerning air
navigation, its infrastructure, flight
inspection, prevention of unlawful
interference, and facilitation of border-
crossing procedures for international civil
aviation.
Source: https://economictimes.indiatimes.com/industry/transportation/airlines-/-
aviation/india-to-join-international-climate-action-in-civil-aviation-from-
2027/articleshow/99870525.cms

Q.36) Solution (c)

Statement Analysis:

Statement 1 Statement 2

Correct Correct

The National Manufacturing Innovation The NMIS survey shows that innovation is not
Survey (NMIS) 2021-22 is a joint study by yet common in manufacturing but has proved
the Department of Science and Technology to be profitable for firms.
(DST) and the United Nations Industrial The NMIS 2021-22 survey had two specific
Development Organization (UNIDO). components: the firm-level survey and the
sectorial systems of innovation (SSI) survey.

https://upscpdf.com/ https://upscpdf.com/ https://upscpdf.com/


https://upscpdf.com/ https://upscpdf.com/ https://upscpdf.com/

.
IASBABA’S PRELIMS CURRENT AFFAIRS TEST- MAY 2023 SOLUTIONS

Its objective is to evaluate the innovation  The Firm-level survey: captured data
performance of manufacturing firms in India. related to types of innovations and
This study is a follow-up to DST’s first National innovative measures taken by firms.
Innovation Survey held in 2011. This includes the process of innovation, access
The survey offered an empirical
to finance, resources, and information for
understanding of current innovation activities innovation, besides also recording the factors
of the manufacturing economy in India as well impacting the innovation activities in a firm.
as ways to navigate organisational rigidity to One in four firms was found to have
facilitate market demand for innovations. successfully implemented an innovation in
the observation period, and over 80% of these
firms benefitted significantly in expanding
markets and production and reducing costs.
 The Sectorial System of Innovation
survey: mapped the manufacturing
innovation system and its enabling
role in achieving innovations in firms.
The SSI study measured the interactions
between stakeholders of the innovation
ecosystem, relative barriers to innovation, as
well as the convergence or divergence of
current policy instruments in select five key
manufacturing sectors important to the
Indian economy – textiles; food & beverage;
automotive; pharma; and ICT.
The findings from the firm-level survey are
captured in the ‘Assessment of Firm-Level
Innovation in Indian Manufacturing’.
Separately, five reports from the study of the
sectorial systems of innovation within five
manufacturing sectors, namely, Automotive,
Pharmaceutical, Textile, Food & Beverages,
and Information & Communication
Technologies (ICT), have been developed.
Source: https://pib.gov.in/PressReleasePage.aspx?PRID=1920510

https://upscpdf.com/ https://upscpdf.com/ https://upscpdf.com/


https://upscpdf.com/ https://upscpdf.com/ https://upscpdf.com/

.
IASBABA’S PRELIMS CURRENT AFFAIRS TEST- MAY 2023 SOLUTIONS

Q.37) Solution (c)

Statement Analysis:

Statement 1 Statement 2

Correct Incorrect

Psychedelics are a group of drugs that alter Psychedelics are non-addictive and non-toxic.
perception, mood, and thought Compared to illicit drugs, psychedelics
processing while a person is still clearly cause much less harm to the end user.
conscious.
Usually, the person’s insight also remains
unimpaired.
Statement 3

Correct

The two most commonly used psychedelics


are d-lysergic acid diethylamide (LSD) and
psilocybin.
Less common ones include mescaline, found
in the North American peyote cactus (
Lophophora williamsii), and N, N-
dimethyltryptamine, the principal component
of the South American ceremonial sacrament
ayahuasca.
In India, the Narcotic Drugs and Psychotropic
Substances Act 1985, prohibits the use of
psychedelic substances.
Ketamine is a dissociative anaesthetic with
psychedelic properties and is used under
strict medical supervision, for anaesthesia
and to treat treatment-resistant depression.
Source: https://www.thehindu.com/sci-tech/science/psychedelic-drugs-recreational-use-
medicinal-use-treatment-resistant-depression/article66789786.ece

https://upscpdf.com/ https://upscpdf.com/ https://upscpdf.com/


https://upscpdf.com/ https://upscpdf.com/ https://upscpdf.com/

.
IASBABA’S PRELIMS CURRENT AFFAIRS TEST- MAY 2023 SOLUTIONS

Q.38) Solution (a)

Statement Analysis:

Statement 1 Statement 2

Correct Correct

Neurotoxins can damage, destroy, or impair Neurotoxins are absorbed through inhalation,
the functioning of the central and/or ingestion, skin contact, or injection and can
peripheral nervous system. have immediate or long-lasting impacts by
Neurotoxins may damage neurons, axons, causing neurons to malfunction or by
and/or glia resulting in loss of specific nuclei disrupting interneuron communication.
and/or axonal tracts or demyelination. Certain neurotoxins are highly potent and
They may also cause metabolic have been developed into chemical weapons.
imbalances that can secondarily affect the Sarin is an organophosphorus compound that
central nervous system (CNS). is classified as a weapon of mass destruction.
Many neurotoxins are of external origin, Neurotoxins such as arsenic, chlorpyrifos,
entering the body from environmental DDT (dichlorodiphenyltrichloroethane), ethyl
sources. alcohol, fluoride, lead, polybrominated
Others, however, are endogenous, being diphenyl ethers (PBDEs), polychlorinated
produced and existing within the body. biphenyls (PCBs), manganese, mercury, and
Examples of endogenous neurotoxins include toluene are major contributors to the
the neurotransmitters nitric oxide and prevalence of neurobehavioral disorders.
glutamate. The impact of some neurotoxins, such as lead
and ethyl alcohol, is well-documented.
Statement 3

Incorrect

Neurotoxins occur synthetically as well as


naturally.

Source: https://indianexpress.com/article/explained/explained-health/ludhiana-gas-leak-
deaths-neurotoxins-explained-8585039/

https://upscpdf.com/ https://upscpdf.com/ https://upscpdf.com/


https://upscpdf.com/ https://upscpdf.com/ https://upscpdf.com/

.
IASBABA’S PRELIMS CURRENT AFFAIRS TEST- MAY 2023 SOLUTIONS

Q.39) Solution (b)

Statement Analysis:

Statement 1 Statement 2

Incorrect Correct

Article 142 deals with the enforcement of Cases where the Supreme Court has invoked
decrees and orders of the Supreme Court. its plenary powers under Article 142:
Article 142 provides discretionary power to  Manohar Lal Sharma v. Principal
the Supreme Court as it states that the Secretary(2014): The Supreme Court
Supreme Court in the exercise of its can deal with exceptional
jurisdiction may pass such decree or make circumstances interfering with the
such order as is necessary for doing complete larger interest of the public in order to
justice in any cause or matter pending before fabricate trust in the rule of law.
it.  A.R. Antulay v. R.S. Nayak(1988): The
Subsection 1 of Article 142 provides a unique Supreme Court held that any
power to the Supreme Court, to do complete discretion which is given by the court
justice between the parties. should not be arbitrary or in any way
According to some eminent jurists, natural be inconsistent with provisions of any
justice is above the law, and the Supreme statute laid down.
Court shall have the full right to pass any order  Union Carbide Corporation v. Union of
that it considers just. India (1989): In the Bhopal Gas
Therefore, the Supreme Court shall exercise Tragedy Case, the court ordered to
these powers and will not be deterred from award of compensation to the victims
doing justice by the provision of any rule or and placed itself in a position above
law, executive practice or executive circular or the Parliamentary laws.
regulation, etc.  Siddiq v. Mahant Suresh
The framers of the Constitution felt that this Das (2019): Popularly known as
provision is of utmost significance to those the Ayodhya dispute, the Supreme
people who have to suffer due to the delay in Court exercised the powers
getting their necessary relief due to the mentioned under Article 142 of the
disadvantaged position of the judicial system. Constitution.
Source: https://indianexpress.com/article/explained/explained-law/supreme-court-divorce-
explained-ruling-8585167/

https://upscpdf.com/ https://upscpdf.com/ https://upscpdf.com/


https://upscpdf.com/ https://upscpdf.com/ https://upscpdf.com/

.
IASBABA’S PRELIMS CURRENT AFFAIRS TEST- MAY 2023 SOLUTIONS

Q.40) Solution (c)

Statement Analysis:

Statement 1 Statement 2

Correct Incorrect

The Golden Globe Race is being conducted by Recently, Abhilash Tomy created history as
the K.’s Sir Robin Knox Johnston. the first Indian to complete the Golden Globe
Its objective is to commemorate the world’s Race.
first solo non-stop circumnavigation
undertaken by him in 1968, onboard the
Indian-built boat Suhaili.
Participants in the GGR are required to sail
around the world, single-handedly and non-
stop.
Statement 3

Correct

The uniqueness of the race is that boat


designs and technology newer than 1968 are
not permitted.
Also, the use of the Global Positioning System
(GPS), satellite communication, and
navigational aids are forbidden in the 30,000-
mile journey.
The sailors each have a satellite
phone and emergency locator beacons to be
used for medical emergencies only.
Entrants are limited to sailing similar yachts,
between 32 ft and 36 ft, designed prior to
1988 with a full-length keel with a rudder.
Equipment similar to what was available to Sir
Robin in that first race was to be used.
16 sailors from 11 countries depart from Les
Sables-d’Olonne, France.
Source: https://indianexpress.com/article/sports/sport-others/abhilash-tomy-completes-
the-golden-globe-race-finishing-second-makes-maritime-adventure-history-8582466/

https://upscpdf.com/ https://upscpdf.com/ https://upscpdf.com/


https://upscpdf.com/ https://upscpdf.com/ https://upscpdf.com/

.
IASBABA’S PRELIMS CURRENT AFFAIRS TEST- MAY 2023 SOLUTIONS

Q.41) Solution (c)

Statement Analysis:

Statement 1 Statement 2

Correct Correct

Bluebugging is a form of hacking that lets Preventive Measures:


attackers access a device through its  Turning off Bluetooth and
discoverable Bluetooth connection. disconnecting paired Bluetooth
A hacker can gain unauthorized access to devices when not in use.
these apps and devices and control them as  Making Bluetooth
per their wish through bluebugging. devices undiscoverable from
Any Bluetooth-enabled device including True Bluetooth settings.
Wireless Stereo (TWS) devices or earbuds  Updating the device’s system software
susceptible to bluebugging. to the latest version.
 Limited use of public Wi-Fi.
 Watch out for suspicious activities on
your device.
 Monitoring of sudden spikes in data
usage.
 Usage of modern anti-virus software.
Source: https://www.thehindu.com/news/national/andhra-pradesh/andhra-pradesh-
beware-of-bluebugging-smartphone-users-cautioned/article66796736.ece

Q.42) Solution (b)

Statement Analysis:

Statement 1 Statement 2

Incorrect Correct

Shilabhattarika was a Sanskrit poetess who The 10th-century poet Rajashekhara praises
lived in the 9th century. Shilabhattarika as a leading figure of the
M. B. Padma, a scholar of the University of Panchali literary style (one of the four major
Mysore, speculates that she may be the same contemporary literary styles)
as Shila-maha Devi, the queen of the 8th- The other three are Vaidharbhi, Gaudi, and
century Rashtrakuta ruler Dhruva. Lati.
His theory is based on the fact that the suffix According to Rajashekhara, the Panchali style
“Bhattarika” attached to the poet’s name can be traced to the works of Shilabhattarika,
indicates her high social status and that the

https://upscpdf.com/ https://upscpdf.com/ https://upscpdf.com/


https://upscpdf.com/ https://upscpdf.com/ https://upscpdf.com/

.
IASBABA’S PRELIMS CURRENT AFFAIRS TEST- MAY 2023 SOLUTIONS

queen is known to have made generous and possibly to some of the works of the 7th-
grants to scholars. century poet Bana.
Shilabhattarika has been quoted by several
classical Sanskrit literary critics, and her
verses appear in most major Sanskrit
anthologies.
Sharngadhara-paddhati, a 14th-century
anthology, praises her and three other female
poets in the following words.
Shilabhattarika, Vijja, Marula, and Morika are
poetesses of renown with great poetic genius
and erudition.
Statement 3

Correct

She is known to have written at least 46


poems on topics such as “love, morality,
politics, nature, beauty, the seasons, insects,
anger, indignation, codes of conduct, and the
characteristic features of various kinds of
heroines.“
However, most of her works are now lost.
Only six of her short poems are extant.
Source: https://www.thehindu.com/news/national/other-states/copper-plates-decoded-by-
pune-based-bhandarkar-institute-sheds-light-on-celebrated-ancient-sanskrit-poetess-
shilabhattarika/article66795789.ece

Q.43) Solution (b)

Statement Analysis:

Statement 1 Statement 2

Incorrect Correct

Yellow Fever is an acute viral haemorrhagic The virus is endemic in tropical areas of Africa
disease transmitted by infected mosquitoes. and Central and South America.
The yellow in the name refers to the jaundice Yellow fever is prevented by an extremely
that affects some patients. effective vaccine which is safe and
affordable.

https://upscpdf.com/ https://upscpdf.com/ https://upscpdf.com/


https://upscpdf.com/ https://upscpdf.com/ https://upscpdf.com/

.
IASBABA’S PRELIMS CURRENT AFFAIRS TEST- MAY 2023 SOLUTIONS

Its symptoms include fever, headache, Eliminate yellow fever epidemics(EYE) 2017-
jaundice, muscle pain, nausea, vomiting, and 2026 has been developed by a coalition of
fatigue. partners (Gavi, UNICEF, and WHO) that aims
A small proportion of patients who contract at ending yellow fever epidemics by 2026.
the virus develop severe symptoms and It is usually compulsory to get vaccinated
approximately half of those die within 7 to 10 before travelling to any of the yellow fever-
days. endemic countries in parts of Africa, and
Central and South America.
Statement 3

Correct

The Yellow fever vaccine is known as 17D.


According to the World Health Organization
(WHO), it is safe and affordable.
However, there are reports of multisystem
organ failure following vaccination.
Source: https://www.thehindu.com/news/national/operation-kaveri-117-passengers-not-
vaccinated-against-yellow-fever-quarantined-on-arrival/article66793656.ece

Q.44) Solution (b)

Statement Analysis:

Statement 1 Statement 2

Incorrect Correct

EX AGNI DAMAN-23 is a Civil-Military mutual Its objectives:


firefighting exercise. to emphasise the need for awareness and
A total of 56 firefighting columns including 32 training of men in firefighting.
from civil agencies like the National Disaster to exercise the capabilities of combined forces
Relief Force (NDRF), State Disaster Relief of all agencies including military and civil.
Force (SDRF), MC Alandi, Fire Department It was conducted at 29 Field Ammunition
Pimpri Chinchwad, PMRDA Akurdi, MIDC Depot, Dehu Road under the aegis
Ambi Talegaon, MC Talegaon Dhabade, Nagar of Headquarters Southern Command in Apr
Parishad Chakan, Main Fire Department 2023.
Bhawani Peth, Tata Motors Ltd, Mahindra
Vehicle Ltd & Bajaj Auto
participated actively in the exercise.

https://upscpdf.com/ https://upscpdf.com/ https://upscpdf.com/


https://upscpdf.com/ https://upscpdf.com/ https://upscpdf.com/

.
IASBABA’S PRELIMS CURRENT AFFAIRS TEST- MAY 2023 SOLUTIONS

Drills and procedures including swift response


strategy against all types of fire were
rehearsed in a coordinated manner.
Source: https://pib.gov.in/PressReleaseIframePage.aspx?PRID=1920627

Q.46) Solution (c)

India’s first cable-stayed rail bridge connecting J&K is ready'


Anji Khad Bridge will be the cable-stayed railway bridge in the country. Hence statement 1 is
correct.
About Anji Khad Bridge:
It is an under-construction rail bridge in Jammu & Kashmir on the Jammu-Baramulla rail line.
It will connect the Kashmir with the rest of India. Hence statement 3 is incorrect.
It is constructed over the Anji River which is a tributary of Chenab. Hence statement 2 is
incorrect.

https://www.hindustantimes.com/india-news/anji-khad-india-s-first-cable-stayed-rail-
bridge-is-ready-says-ashwini-vaishnaw-101682770795568.html

Q.47) Solution (b)

Statement Analysis:

After a six-year absence, the lesser flamingos, resplendent with deep red legs and bills, have
finally found their way back to Pulicat Lake.

About Lesser Flamingo:

They are the smallest of all flamingos but have the largest population. Hence statement 1 is
incorrect.

Description They have pale pink plumage, legs, and bills. They possess the “hallux” or “hind
toe” that some other flamingos do not have. They are the smallest and brightest of all
flamingos.

Size approx. 80-90 cm long. The males are a little taller than the females.

Diet Their diet consists entirely of microscopic blue-green algae and benthic diatoms found
only in alkaline salt lakes, salt pans, estuaries, and salt lagoons. Hence statement 2 is correct.

Habitat It is in habitat coastal and inland wetlands.

https://upscpdf.com/ https://upscpdf.com/ https://upscpdf.com/


https://upscpdf.com/ https://upscpdf.com/ https://upscpdf.com/

.
IASBABA’S PRELIMS CURRENT AFFAIRS TEST- MAY 2023 SOLUTIONS

Geographical distribution These birds are primarily found in eastern and southern Africa,
especially Sub-Saharan Africa, South Asia, and North Africa.

IUCN Status Ne1ar Threatened. Hence Statement 3 is incorrect.

CITES Appendix II

Fun Fact The name flamingo is originally derived from the Portuguese language and means
"red goose," which is a reference to their flying formation and the noise they make.

https://www.thehindu.com/sci-tech/energy-and-environment/a-flamboyance-of-lesser-
flamingos-have-arrived-at-chennais-pulicat/article66776533.ece

Q.48) Solution (d)

Statement Analysis:

After 110 years: Tiger spotted at Kalesar National Park.

The Haryana Forest Department has launched efforts to trace pug marks of a tiger that was
caught in a camera trap at Kalesar National Park in Yamunanagar — the first sighting of the
big cat in any part of the state in 110 years.

About Kalesar National Park:

 Kalesar Nation Park is in the Yamunanagar district of Haryana. Hence Statement 1 is


correct.
 It was declared a National Park on 8th December 2003.
 Kalesar National Park shares a corridor with Himachal Pradesh's Simbalbara National
Park and Uttarakhand's Rajaji National Park. Hence Statement 3 is correct.
 The park is named after a temple known as Kalesar Mahadev Temple which is located
inside its premises. Hence Statement 2 is correct.

https://www.hindustantimes.com/cities/chandigarh-news/tiger-captured-on-camera-after-
110-years-in-haryana-s-kalesar-national-park-wildlife-officials-elated-tiger-
kalesarnationalpark-haryana-wildlife-101682626180634.html

https://upscpdf.com/ https://upscpdf.com/ https://upscpdf.com/


https://upscpdf.com/ https://upscpdf.com/ https://upscpdf.com/

.
IASBABA’S PRELIMS CURRENT AFFAIRS TEST- MAY 2023 SOLUTIONS

Q.49) Solution (a)

Statement Analysis:

Statement 1 Statement 2

Correct Correct

King Cobra is one of the most venomous They live mainly in the rainforests and plains
snakes on the planet. of India, southern China, and Southeast Asia.
It is also the longest of all venomous snakes. They are comfortable in a variety of habitats,
The venom is not the most potent among including forests, bamboo thickets, mangrove
venomous snakes, but the amount of swamps, high-altitude grasslands, and in
neurotoxin they can deliver in a single bite up rivers.
to two-tenths of a fluid ounce is enough to kill
20 people or even an elephant.
They are the only snakes in the world that
build nests for their eggs, which they guard
ferociously until the hatchlings emerge.
Statement 3

Incorrect

Protection Status:
 IUCN Red List: Vulnerable.
 CITES: Appendix II.
 Wildlife (Protection) Act, 1972:
Schedule II.
Source: https://timesofindia.indiatimes.com/topic/king-cobra-snake/news

Q.50) Solution (c)

Statement Analysis:

Statement 1 and 2 Statement 3

Correct Correct

The mridangam is the classical drum of South The body of the mridangam is scooped out of
Indian music. a single block of wood.
It is also known by the name of Medal or Jackwood or redwood is the ideal choice of
Maddalam. mridangam makers, but the wood of the
It is one of the oldest Indian percussion morogosa tree or the core of the coconut tree
instruments, originating 2,000 years ago.

https://upscpdf.com/ https://upscpdf.com/ https://upscpdf.com/


https://upscpdf.com/ https://upscpdf.com/ https://upscpdf.com/

.
IASBABA’S PRELIMS CURRENT AFFAIRS TEST- MAY 2023 SOLUTIONS

This traditional instrument is found in various and the palm tree is also used for this
parts of South India. purpose.
It is a popular bifacial drum of Carnatic
music and is used as an accompaniment
in South Indian Classical music.
A similar instrument, the pakhavaj, is played
in the Hindustani tradition of northern India,
as well as in Pakistan and Bangladesh.
Source: https://www.thehindu.com/news/national/tamil-nadu/mridhangam-artiste-
karaikudi-mani-no-more/article66811456.ece

Q.51) Solution (b)

Statement Analysis:

Statement 1 Statement 2

Correct Incorrect

India is the second major producer of India ranks 1st in inland capture fish
fish through aquaculture in the world. production and 3rd in overall fish
India is the 4th largest exporter of fish in the production in the world.
world as it contributes 7.7% to global fish Currently, this sector provides livelihood to
production. more than 2.8 crore people within the
country.
Statement 3

Incorrect

Sagar Parikrama is a navigation journey to be


conducted in all coastal states/UTs through a
pre-decided sea route to demonstrate
solidarity with all fisherfolk, fish farmers, and
concerned stakeholders.
It will focus on the sustainable balance
between the utilisation of marine
fisheries resources for the food security of
the nation and livelihoods of coastal fisher
communities and the protection of marine
ecosystems.

https://upscpdf.com/ https://upscpdf.com/ https://upscpdf.com/


https://upscpdf.com/ https://upscpdf.com/ https://upscpdf.com/

.
IASBABA’S PRELIMS CURRENT AFFAIRS TEST- MAY 2023 SOLUTIONS

Phases of Sagar Parikrama:

 Phase I: The journey covered three


locations in Gujarat - Mandavi, Okha-
Dwarka, and Porbandar.
 Phase II: Seven locations were
covered in Mangrol, Veraval, Diu,
Jafrabad, Surat, Daman, and Valsad.
 Phase III: Coastal areas of
northern Maharashtra, including
Satpati, Vasai, Versova, New Ferry
Wharf (Bhaucha Dhakka), and Sasson
Dock in Mumbai, were part of this
phase.
 Phase IV: Udupi and Dakshina
Kannada districts in Karnataka were
covered during this phase.
 Upcoming Phase V: Phase V of Sagar
Parikrama will cover six locations:
Raigad, Ratnagiri, and Sindhudurg
Districts in Maharashtra, and Vasco,
Maorugoa, and Canacona in Goa.
Note: Andhra Pradesh is the largest producer of fish in India followed by West Bengal.
Source: https://pib.gov.in/PressReleaseIframePage.aspx?PRID=1924524

Q.52) Solution (d)

Statement Analysis:

Statement 1 Statement 2

Incorrect Incorrect

The Internal Displacement Monitoring Centre The number of people living in internal
has released a report titled- The Global displacement reached a record high of 71.1
Report on Internal Displacement 2023 (GRID- million people across 110 countries and
2023). territories.
The IDMC is the world's leading source of 62.5 million as a result of conflict and
data and analysis on internal displacement. It violence, and 8.7 million as a result of
provides high-quality data, analysis, and disasters.
expertise on Internal Displacement with the

https://upscpdf.com/ https://upscpdf.com/ https://upscpdf.com/


https://upscpdf.com/ https://upscpdf.com/ https://upscpdf.com/

.
IASBABA’S PRELIMS CURRENT AFFAIRS TEST- MAY 2023 SOLUTIONS

aim of informing policy and operational Disasters displaced 8.7 million people
decisions that can reduce the risk of future internally in 88 countries and territories as of
displacement. December 2022.
This led to record levels of flood displacement
Country-wise picture: in countries including Pakistan, Nigeria, and
Brazil.
Pakistan had the highest number of disaster By 2021, 30.7 million new displacements
displacements in the world in 2022, at 8.16 were due to disasters. In 2022 some 150
million. countries/territories reported such
In Pakistan, floods displaced millions, displacement.
accounting for a quarter of the global disaster
displacements.
The Philippines was at second rank and
reported 5.44 million displacements and
China at third rank with 3.63 million.
India recorded the fourth largest disaster
displacement, with 2.5 million displacements,
and Nigeria at fifth rank with 2.4 million.
Source: https://www.downtoearth.org.in/news/climate-change/unusually-long-la-ni-a-
displaced-record-number-of-people-in-2022-89247

Q.53) Solution (c)

Statement Analysis:

Statement 1 Statement 2

Incorrect Correct

Sea butterflies, scientific name Thecosomata, Sea butterflies are holoplanktonic (organisms
are a suborder of sea snails known as shelled that pass their whole life floating, drifting, or
pteropods. swimming weakly in the water) and spend
They have muscular feet that allow them to their entire life cycle in the water column.
swim in water instead of gliding on solid
surfaces.
Statement 3 Statement 4

Correct Correct

https://upscpdf.com/ https://upscpdf.com/ https://upscpdf.com/


https://upscpdf.com/ https://upscpdf.com/ https://upscpdf.com/

.
IASBABA’S PRELIMS CURRENT AFFAIRS TEST- MAY 2023 SOLUTIONS

Sea butterflies are found in all oceans but are Sea butterflies play a key role in transporting
more diverse and abundant in colder waters. carbon from the surface to the deep ocean
Sea butterflies have bilateral symmetry and through their shells and fecal pellets.
coiled or uncoiled shells of various shapes and They are a major food source for many fish,
sizes. seabirds, whales, and other marine animals.
Their shell is mostly transparent and very
fragile and can be easily dissolved by ocean
acidification.
They have a pair of wing-like lobes or
parapodia for propulsion and a head with
eyes, tentacles, and a mouth with a long
proboscis to capture prey.
They have a reduced or absent gill and rely on
their body surface for gas exchange.
Source: https://www.downtoearth.org.in/news/world/sea-butterflies-are-threatened-by-
climate-change-here-s-how-this-can-impact-antarctic-marine-ecosystems-
89358#:~:text=As%20the%20sea%20absorbs%20an,these%20delicate%20species%20to%20
survive.

Q.54) Solution (a)

Statement Analysis:

Statement 1 Statement 2

Correct Correct

Artificial sweeteners are sugar Some examples of artificial sweeteners


substitutes that are used as alternatives to are saccharin, aspartame, acesulfame
natural sugars. potassium (Ace-K), sucralose, neotame, and
These sweeteners are chemically synthesized advantame.
and provide a sweet taste without the high-
calorie content of regular sugar.
They are commonly used in various food and
beverage products, including diet sodas,
sugar-free desserts, and low-calorie snacks.
Statement 3

Incorrect

https://upscpdf.com/ https://upscpdf.com/ https://upscpdf.com/


https://upscpdf.com/ https://upscpdf.com/ https://upscpdf.com/

.
IASBABA’S PRELIMS CURRENT AFFAIRS TEST- MAY 2023 SOLUTIONS

Artificial sweeteners offer benefits for weight


management, diabetes control, and tooth
decay prevention, and provide safe options
for individuals with phenylketonuria (PKU), a
genetic disorder, due to their low or zero-
calorie content, minimal impact on blood
sugar levels, non-fermentable nature, and
absence of phenylalanine.
Source: https://indianexpress.com/article/explained/explained-health/artificial-sweetners-
harmful-who-recommendations-8614073/

Q.55) Solution (b)

Statement Analysis:

Statement 1 Statement 2

Incorrect Correct

iDEX, launched in 2018, is an ecosystem to It provides funding/grants to Micro Small and


foster innovation & technology development Medium Enterprises (MSMEs), start-ups,
in Defence and Aerospace by engaging individual innovators, R&D (Research and
innovators & entrepreneurs to deliver Developments) institutes, and academia to
technologically advanced Solutions for carry out research and development.
modernizing the Indian Military.
The iDEX-Prime aims to support projects
requiring support beyond Rs 1.5 crore up to
Rs 10 crore, to help ever-growing start-ups in
the defence sector.
iDEX portal was launched to provide wider
publicity and better visibility of iDEX activities
and enable more efficient running of future
challenges through better information
management.
Statement 3

Correct

iDEX is funded and managed by Defence


Innovation Organisation (DIO).

https://upscpdf.com/ https://upscpdf.com/ https://upscpdf.com/


https://upscpdf.com/ https://upscpdf.com/ https://upscpdf.com/

.
IASBABA’S PRELIMS CURRENT AFFAIRS TEST- MAY 2023 SOLUTIONS

iDEX will function as the executive arm of DIO,


carrying out all the required activities while
DIO will provide high-level policy guidance to
iDEX.
Note:
 Mission DefSpace was inaugurated by the Prime Minister of India during the October
2022 edition of India’s DefExpo.
 It aims to nurture the Indian Private Space industry through challenges addressing
every stage of a space mission – from mission planning to satellite data analytics.
 This mission encompasses 75 Defense Space Challenges that directly suit the needs of
end users.
Source: https://economictimes.indiatimes.com/news/defence/defence-ministrys-idex-
achieves-another-milestone-with-signing-of-its-250th-contract/articleshow/100258099.cms

Q.56) Solution (b)

Statement Analysis:

Statement 1 Statement 2

Incorrect Correct

The AePS is a bank-led model that The AePS eliminates the need for OTPs, bank
allows online interoperable financial account details, and other financial
transactions at Point of Sale (PoS) or micro- information.
ATMs through the Business Correspondent Transactions can be carried out with only
(BC) of any bank using the Aadhaar the bank name, Aadhaar number, and
authentication. captured fingerprint during Aadhaar
It was taken up by the National Payments enrollment.
Corporation of India (NPCI) - a joint initiative
of the Reserve Bank of India (RBI) and the
Indian Banks’ Association (IBA).
The AePS is meant to provide easy and secure
access to banking services for the poor and
marginalized sections of society, especially in
rural and remote areas.
Statement 3

Correct

https://upscpdf.com/ https://upscpdf.com/ https://upscpdf.com/


https://upscpdf.com/ https://upscpdf.com/ https://upscpdf.com/

.
IASBABA’S PRELIMS CURRENT AFFAIRS TEST- MAY 2023 SOLUTIONS

The AePS helps in deepening social security by


facilitating cash transfers from various
government schemes such as PM-
KISAN, MGNREGA, etc., directly into the
beneficiaries' bank accounts.
The AePS enables interoperability among
different banks and financial
institutions, allowing customers to access
their bank accounts through any BC or micro-
ATM of any bank.
Source: https://www.thehindu.com/sci-tech/technology/explained-gaps-aadhaar-enabled-
payment-system-aeps-abused-cybercriminals/article66842275.ece

Q.57) Solution (b)

Statement Analysis:

Statement 1 Statement 2

Incorrect Correct

The Indian Ocean Conference is a flagship The 6th Indian Ocean Conference was held in
consultative forum of the Indian Ocean Dhaka, Bangladesh.
countries to deliberate upon the prospects of Its theme was "Peace Prosperity and
regional cooperation for Security and Growth Partnership for a Resilient Future", bringing
for All in the Region (SAGAR). together delegates from over 25 countries to
The first edition of the Indian Ocean discuss ways to promote economic
conference was held in Singapore in 2016 and development while maintaining peace and
the fifth in 2021 in Abu Dhabi, United Arab stability in the region.
Emirates.
Source: https://indianexpress.com/article/india/6th-indian-ocean-conference-better-
connectivity-respecting-sovereignty-should-be-priority-says-jaishankar-in-dhaka-8606508/

https://upscpdf.com/ https://upscpdf.com/ https://upscpdf.com/


https://upscpdf.com/ https://upscpdf.com/ https://upscpdf.com/

.
IASBABA’S PRELIMS CURRENT AFFAIRS TEST- MAY 2023 SOLUTIONS

Q.58) Solution (c)

Statement Analysis:

Statement 1 Statement 2

Correct Correct

Land reclamation refers to the process of  Coastal wetlands, such


creating new land by altering the topography as mangroves, salt marshes,
of existing bodies of water, such as seas, and estuaries, are highly productive
rivers, lakes, or marshes. ecosystems that provide numerous
It is typically done along coastlines but can ecological benefits.
also occur inland, involving the conversion  Land reclamation often
of wetlands or other water bodies. involves draining or filling these
Land reclamation has a long history of being wetlands, resulting in their
used to expand coastal areas destruction or alteration.
for agriculture and industrial purposes.  This loss of wetlands can disrupt the
natural balance of coastal ecosystems,
Traditionally, land reclamation meant affecting water quality, fish nurseries,
building a series of dikes to enclose tidal and the overall resilience of the
marshes or shallow offshore waters and coastal zone.
draining these enclosures to create dry land.  Most coastal land expansion in the
past couple of decades happened
Today, major engineering projects involve in low-lying areas, with more
the construction of kilometres of offshore than 70% of that land “at high risk
concrete barrier walls, which are filled with from coastal flooding between 2046
substantial amounts of sand, earth, clay, or and 2100,” due in part to storm surges
rock, often shipped in from far afield. linked to global warming and the risk
of land subsidence.
 Stronger storms and increasingly
destructive flooding are already
taking coastal communities by
surprise.
 Using materials like sand
 which are obtained from
the marine and river environment can
mean the destruction of habitats and
spawning grounds of organisms.
 Several countries have already
banned the export of sand for land
reclamation. The resulting sand

https://upscpdf.com/ https://upscpdf.com/ https://upscpdf.com/


https://upscpdf.com/ https://upscpdf.com/ https://upscpdf.com/

.
IASBABA’S PRELIMS CURRENT AFFAIRS TEST- MAY 2023 SOLUTIONS

shortage has forced some


construction companies to extract
sand and clay from the ocean floor,
destroying the seabed ecosystem in
the process.
Source: https://indianexpress.com/article/explained/explained-climate/sea-levels-rise-land-
reclamation-8610074/

Q.59) Solution (d)

Statement Analysis:

Statement 1 Statement 2

Correct Correct

Transformers are a type of deep learning Transformers consist of an encoder and a


model used for natural language processing decoder, which work together to process
(NLP) and computer vision (CV) tasks. input and generate output.
They utilize a mechanism called “self- The encoder converts words into abstract
attention” to process sequential input data. numerical representations and stores them in
Transformers can process the entire input a memory bank.
data at once, capturing context and The decoder generates words one by one,
relevance. referring to the generated output and
They can handle longer sequences efficiently consulting the memory bank through
and overcome the vanishing gradients attention.
problem faced by recurrent neural networks
(RNNs).
Transformers were introduced in 2017
through the paper "Attention is All You
Need" by Google Brain.
Statement 3

Correct

Transformers have revolutionized tasks such


as language translation, sentiment analysis,
text summarization, and natural language
understanding.

https://upscpdf.com/ https://upscpdf.com/ https://upscpdf.com/


https://upscpdf.com/ https://upscpdf.com/ https://upscpdf.com/

.
IASBABA’S PRELIMS CURRENT AFFAIRS TEST- MAY 2023 SOLUTIONS

They process entire sentences or paragraphs,


capturing intricate linguistic patterns and
semantic meaning.
Transformers have made significant strides in
computer vision tasks, surpassing traditional
convolutional neural networks (CNNs).
They analyze images by breaking them into
patches and learning spatial
relationships, leading to improved image
classification, object detection, and more.
Note: Machine learning is a branch of artificial intelligence that involves developing
algorithms that can learn and improve from data. It enables computers to make
predictions or take actions without being explicitly programmed. It uses statistical techniques
and algorithms to analyze and interpret complex data sets.
Source: https://www.thehindu.com/sci-tech/science/transformer-machine-learning-model-
chatgpt-explained/article66829325.ece

Q.60) Solution (b)

Statement Analysis:

 Alzheimer's disease is a progressive neurodegenerative disorder that affects


the brain, leading to memory loss, cognitive decline, behavioral changes, problems
with words in speaking or writing, poor judgment, changes in mood and personality,
confusion with time or place, etc.
 Donanemab belongs to a class of antibody-based therapies that target amyloid-beta
(Aβ) proteins. These proteins can form amyloid plaques in the brain, leading to
cognitive decline.
 Donanemab aims to remove these plaques and slow the progression of the disease.
 Lecanemab belongs to a class of drugs called monoclonal antibodies. These antibody-
mediated drugs also target beta-amyloids and disrupt cell function.
Source: https://indianexpress.com/article/explained/explained-health/new-alzheimer-
drugs-donanemab-lecanemab-comparison-8596341/

Q.61) Solution (a)

Statement Analysis:

A hysterectomy is a surgical procedure that involves the removal of the uterus (womb), the
organ in a woman's body where a baby develops during pregnancy.
Types:

https://upscpdf.com/ https://upscpdf.com/ https://upscpdf.com/


https://upscpdf.com/ https://upscpdf.com/ https://upscpdf.com/

.
IASBABA’S PRELIMS CURRENT AFFAIRS TEST- MAY 2023 SOLUTIONS

 When only the uterus is removed, it is called a partial hysterectomy.


 When the uterus and cervix are removed, it is called a total hysterectomy.
 When the uterus, cervix, part of the vagina, and a wide area of ligaments and tissues
around these organs are removed, it is called a radical hysterectomy.
Source: https://www.thehindu.com/sci-tech/health/centre-seeks-hysterectomy-data-from-
all-states/article66860310.ece

Q.62) Solution (c)

Statement Analysis:

Statement 1 Statement 2

Correct Correct

The program of People’s Biodiversity Register PBR is also a mechanism to create awareness
(PBR) is designed as a tool for the formal among the people about the condition of
maintenance of the local knowledge of plants and animals and their conservation and
biodiversity. sustainable utilization.
PBR is a record of knowledge, perception, and This mechanism can bring people to
attitude of people about natural resources, participate in development planning which
plants, and animals, their utilization, and would be ecologically sustainable and socially
conservation in a village or a panchayat. justifiable.
Source: https://pib.gov.in/PressReleseDetailm.aspx?PRID=1926651

Q.63) Solution (b)

Statement Analysis:

Statement 1 Statement 2

Incorrect Correct

The International Museum Expo 2023 Mascot The Dancing Girl was discovered in one such
was a life-size (5 ft as compared to the original excavation in 1926, by British archaeologist
10 cm) figure inspired by the Dancing Girl of Ernest McKay in Mohenjo-Daro’s citadel.
the Indus Valley Civilization. Dancing Girl is a prehistoric bronze
The traditional craft of Channapatna toys was sculpture made in lost-wax casting during c.
used to create this mascot. 2300–1750 BC.
International Museum Day is an international It is 10.5 cm in height, 5 cm in width, and 2.5
day held annually on or around 18 May, cm in depth.

https://upscpdf.com/ https://upscpdf.com/ https://upscpdf.com/


https://upscpdf.com/ https://upscpdf.com/ https://upscpdf.com/

.
IASBABA’S PRELIMS CURRENT AFFAIRS TEST- MAY 2023 SOLUTIONS

coordinated by the International Council of Even though Mohenjodaro and Harappa


Museums. became part of Pakistani territory after the
The event highlights a specific theme, which Partition, the Dancing Girl remained in India
changes every year reflecting a relevant as part of an agreement.
theme or issue facing museums Presently, the bronze figurine is on display in
internationally. the Indus Valley Civilization gallery in the
National Museum of India, New Delhi.

Source: https://indianexpress.com/article/explained/explained-history/what-
mohenjodaros-dancing-girl-figurine-tells-us-about-the-prehistoric-civilisation-8627594/

Q.64) Solution (b)

Statement Analysis:

Statement 1 Statement 2

Incorrect Correct

The ‘Sengol’ originated from the Chola The ‘Sengol’ was used as a symbol of power
dynasty which was one of the most important transfer from one king to his successor.
and powerful dynasties in the history of India. It represents the value of fair and equitable
The Sengol is a scepter made of gold and governance.
silver and is decorated with many precious The ‘Sengol’ was received by Independent
stones. India’s first Prime Minister, Jawaharlal Nehru,
It is 5 feet long and carries a golden orb at the to symbolically represent the transfer of
top. The orb has a carving of Nandi, the bull power from the British.
that is precious to Lord Shiva. Thereafter, it was kept in the Nehru Gallery of
The Sengol is a powerful symbol of the Chola the Allahabad Museum.
Kings' authority and their commitment to
justice.
The sceptre is a reminder of India’s rich
history and culture.

https://upscpdf.com/ https://upscpdf.com/ https://upscpdf.com/


https://upscpdf.com/ https://upscpdf.com/ https://upscpdf.com/

.
IASBABA’S PRELIMS CURRENT AFFAIRS TEST- MAY 2023 SOLUTIONS

Source: https://www.thehindu.com/news/national/inspired-by-cholas-handed-over-by-
british-to-nehru-historic-sengol-to-be-installed-in-new-parliament-
building/article66888185.ece

Q.65) Solution (a)

Statement Analysis:

Statement 1 Statement 2

Correct Incorrect

“Forum shopping” is a term that describes The concept of forum shopping has not been
the strategy of some litigants or lawyers who defined in any Indian statute.
try to find the most favourable court or judge However, Indian Judiciary through its
for their case. observation has assisted in streamlining this
Litigants or lawyers do this by looking at concept in the country’s legal system.
various factors, such as the reputation, law, The practice of forum shopping is not
or procedure of the court or judge, and the permitted by Indian law.
likelihood of getting a positive outcome.
This practice undermines the integrity and
impartiality of the judicial system, as well as
the merit of judgments of the judicial system.
Moreover, it can also lead to wastage of
judicial resources, delay in justice delivery,
and inconsistency in legal precedents by
resorting to such practises to get favourable
judgements.
Note: SC Bench of Justice S. Abdul Nazeer and Justice Krishna Murari in the case of ‘Vijay
Kumar Ghai vs. State of W.B- termed forum shopping as a “disreputable practise by the
courts” that “has no sanction and paramountcy in law”.
Source: https://indianexpress.com/article/explained/explained-law/cji-chandrachud-forum-
shopping-sc-8624879/

https://upscpdf.com/ https://upscpdf.com/ https://upscpdf.com/


https://upscpdf.com/ https://upscpdf.com/ https://upscpdf.com/

.
IASBABA’S PRELIMS CURRENT AFFAIRS TEST- MAY 2023 SOLUTIONS

Q.66) Solution (d)

Statement Analysis:

 India and Australia signed the Migration and Mobility Partnership Agreement.
 India has similar mobility agreements with Austria, France, the United Kingdom,
Germany, and Finland.
 The agreement is aimed at making it easier for students, academics, and professionals
to live study, and work in each other’s countries.
 It will regulate multiple entry visas for professionals and student exchange programs.
 These programs will be reviewed regularly by a Joint Working Group (JWG) to ensure
that they are meeting their objectives and delivering the desired outcomes.
Source: https://www.hindustantimes.com/india-news/india-and-australia-sign-migration-
and-mobility-pact-discuss-critical-minerals-and-renewable-energy-cooperation-ceca-to-be-
concluded-by-yearend-101684953723402.html

Q.67) Solution (b)

Statement Analysis:

Statement 1 Statement 2

Incorrect Correct

Article 123 of the Constitution of India grants The Governor of a state can also issue
the President certain law-making powers to ordinances under Article 213 of the
promulgate ordinances when either of the Constitution of India when the state
two Houses of Parliament is not in session, in legislative assembly is not in session.
urgent situations. If the two Houses start their sessions on
Hence, it is not possible for the ordinances to different dates, the later date is considered.
be issued by Parliament.
When an ordinance is promulgated but
the legislative session is yet to commence,
the ordinance remains in effect as law. It has
the same force and effect as an Act of the
legislature.
But it requires subsequent ratification by
Parliament within six weeks of its reassembly.
An ordinance promulgated by the President
has a maximum validity of six months and six
weeks from the date of its promulgation.

https://upscpdf.com/ https://upscpdf.com/ https://upscpdf.com/


https://upscpdf.com/ https://upscpdf.com/ https://upscpdf.com/

.
IASBABA’S PRELIMS CURRENT AFFAIRS TEST- MAY 2023 SOLUTIONS

Source: https://indianexpress.com/article/explained/explained-law/govts-power-to-
promulgate-repromulgate-ordinances-why-how-8625383/

Q.68) Solution (d)

Statement Analysis:

Statement 1 Statement 2

Correct Correct

Small modular reactors (SMRs) are advanced Many of the benefits of SMRs are inherently
nuclear reactors that have a power capacity linked to the nature of their design – small
of up to 300 MW (e) per unit, which is about and modular.
one-third of the generating capacity of · Given their smaller size, SMRs can be
traditional nuclear power reactors. sited in locations not suitable for larger
SMRs, which can produce a large amount of nuclear power plants.
low-carbon electricity, are: · SMRs offer savings in cost and
 Small– physically a fraction of the size construction time, and they can be deployed
of a conventional nuclear power incrementally to match increasing energy
reactor. demand.
 Modular – making it possible for
systems and components to be
factory-assembled and transported as
a unit to a location for installation.
 Reactors – harnessing nuclear fission
to generate heat to produce energy.
Statement 3

Correct

In areas lacking sufficient lines of


transmission and grid capacity, SMRs can be
installed into an existing grid or remotely off-
grid, as a function of its smaller electrical
output, providing low-carbon power for
industry and the population.
SMRs have reduced fuel requirements. Power
plants based on SMRs require less frequent
refuelling, every 3 to 7 years, in comparison

https://upscpdf.com/ https://upscpdf.com/ https://upscpdf.com/


https://upscpdf.com/ https://upscpdf.com/ https://upscpdf.com/

.
IASBABA’S PRELIMS CURRENT AFFAIRS TEST- MAY 2023 SOLUTIONS

to between 1 and 2 years for conventional


plants.
Some SMRs are designed to operate for up to
30 years without refuelling.
Source: https://economictimes.indiatimes.com/industry/energy/power/india-working-on-
small-modular-reactors-jitendra-singh/articleshow/100448604.cms

Q.69) Solution (b)

Statement Analysis:

Statement 1 Statement 2

Incorrect Correct

IIB has been established by the insurance The IIB performs various functions such as:
regulator, Insurance Regulatory and  Providing analytical insights and
Development Authority of India (IRDAI) in the reports to IRDAI and other
year 2009, as a data repository and analytics stakeholders on various aspects of the
body. insurance industry such as trends,
It was registered as an independent nonprofit patterns, performance, risks, frauds,
earning society on 21st November etc.
2012 under the Andhra Pradesh Societies  Developing and maintaining
Registration Act 2001. databases and repositories of
IIB was mandated to be market insurance data such as policy details,
neutral and provide independent and non- claims data, premium data, etc.
judgmental analysis to all stakeholders.  Facilitating data sharing and exchange
It acts as the Sole Repository and Analytics among insurers, intermediaries,
Body for the entire Insurance Sector in India. regulators, and other entities involved
in the insurance ecosystem.
 Developing and implementing
standards and best practices for data
quality, security, and governance in
the insurance sector.
Statement 3

Correct

IIB handles the Central Index Server, which


acts as a nodal point between different
Insurance Repositories and helps in the de-

https://upscpdf.com/ https://upscpdf.com/ https://upscpdf.com/


https://upscpdf.com/ https://upscpdf.com/ https://upscpdf.com/

.
IASBABA’S PRELIMS CURRENT AFFAIRS TEST- MAY 2023 SOLUTIONS

duplication of demat accounts at the stage of


creation of a new account.
The Central Index Server also acts as an
exchange for transmission/routing of
information pertaining to transactions on
each policy between an insurer and the
insurance repository.
The Bureau also maintains a Registry of all
Hospitals in the network of Insurance
(ROHINI). A portal on “Health Portability” for
the use of the Insurers is also in the
production stage.
The Bureau has also been entrusted by IRDAI,
with the responsibility of building a repository
of all Insurance Sales Persons. De-duplication
is also facilitated to ensure that the Applicant
is not engaged with any other insurer or
insurer intermediary.
Source: https://timesofindia.indiatimes.com/city/hyderabad/russian-hackers-carry-out-
ransomware-attack-iib-hit/articleshow/100433653.cms?from=mdr

Q.70) Solution (b)

Statement Analysis:

Statement 1 Statement 2

Incorrect Correct

INS Sindhuratna (S59) (Jewel of the Sea) is Sindhughosh-class submarines are Kilo-class
a Sindhughosh-class diesel-electric diesel-electric submarines in active service
submarine of the Indian Navy. with the Indian Navy.
Eight of the Navy’s diesel-electric The Kilo Class is the NATO designation for a
submarines are Kilo class or Sindhughosh naval diesel-electric submarine made in
class from Russia, sourced between 1984 and Russia.
2000 and INS Sindhuratna is among the oldest The Kilo Class submarines are mainly
Kilo-class submarines in the fleet. intended for anti-shipping and anti-
It was commissioned into the Indian Navy in submarine operations in relatively shallow
December 1988. waters.
The Kilo-class submarines have a
displacement of 2,300 tonnes, a maximum

https://upscpdf.com/ https://upscpdf.com/ https://upscpdf.com/


https://upscpdf.com/ https://upscpdf.com/ https://upscpdf.com/

.
IASBABA’S PRELIMS CURRENT AFFAIRS TEST- MAY 2023 SOLUTIONS

diving depth of 300 metres, and a top speed


of 18 knots. They can operate solo for 45 days
with a crew of over 50.
Source: https://www.thehindu.com/news/national/kilo-class-submarine-ins-sindhuratna-
reaches-india-after-major-refit-in-russia/article66881333.ece

Q.71) Solution (b)

Statement Analysis:

Statement 1 Statement 2

Incorrect Correct

Bara-lacha la, also known as Bara-lacha Pass The two headwaters of the Chenab
is a high mountain pass in the Zanskar range. River, Chandra, and Bhaga, originate near the
It is at an elevation of 4.890m (16,043ft) Baralacha Pass.
above sea level, located in the Lahaul and The Bhaga River originates from Surya taal
Spiti district of Himachal Pradesh, India. lake, which is situated a few kilometres from
It connects the Lahaul district in Himachal the pass.
Pradesh to the Leh district in Ladakh, situatedThe Chandra River originates from a glacier in
along the Leh–Manali Highway. this region. The native name of Chenab,
“Chandrabhaga”, represents the union of the
Chandra and Bhaga rivers downstream.
Source: https://theprint.in/india/border-roads-organisation-rescues-over-70-people-
stranded-at-hps-baralacha-la-pass/1585925/

Q.72) Solution (d)

Statement Analysis:

 G7 stands for “Group of Seven” industrialized nations.


 The bloc meets annually to discuss issues of common interest like global economic
governance, international security, and energy policy.
 Headquarters: The G-7 does not have a formal constitution or a fixed headquarters.
The decisions taken by leaders during annual summits are non-binding.
 G-7 countries include the United States, Canada, France, Germany, Italy, Japan, and
the United Kingdom.
 Members share common values like democracy, respect for human rights and
fundamental freedoms, free markets, and respect for international law.

https://upscpdf.com/ https://upscpdf.com/ https://upscpdf.com/


https://upscpdf.com/ https://upscpdf.com/ https://upscpdf.com/

.
IASBABA’S PRELIMS CURRENT AFFAIRS TEST- MAY 2023 SOLUTIONS

Source: https://indianexpress.com/article/explained/explained-global/g7-talks-about-de-
risking-in-reference-to-china-what-does-this-mean-8623161/

Q.73) Solution (c)

Statement Analysis:

Statement 1 Statement 2

Correct Correct

India established the Forum for FIPIC as a part Forum for India-Pacific Islands Cooperation is
of the Act East Policy. a multinational grouping developed in 2014
Indian Prime Minister Narendra Modi in Fiji for cooperation between India and 14 Pacific
convened FIPIC’s inaugural gathering in 2014. Islands nations.
All 14 countries actively participated in this FIPIC consists of Cook Islands, Fiji, Kiribati,
landmark event. Marshall Islands, Federated States of
In 2015, the second meeting of the forum was Micronesia, Nauru, Niue, Samoa, Solomon
held in Jaipur, with the participation of all 14 Islands, Palau, Papua New Guinea, Tonga,
countries. Tuvalu, and Vanuatu.
The third summit of India-Pacific Islands Co-
operation (FIPIC Summit 2023) held at Port
Moresby was jointly hosted with Papua New
Guinea.
The Prime Minister of India co-chaired the
summit with the PM of Papua New Guinea.
On this occasion, the Prime Minister of India
unveiled a comprehensive 12-step initiative
aimed at advancing India’s collaborations
with the countries in the Pacific region.
Source: https://www.thehindu.com/news/national/pm-modi-underlines-importance-of-
free-and-open-indo-pacific-at-fipic-summit-in-papua-new-guinea/article66879690.ece

https://upscpdf.com/ https://upscpdf.com/ https://upscpdf.com/


https://upscpdf.com/ https://upscpdf.com/ https://upscpdf.com/

.
IASBABA’S PRELIMS CURRENT AFFAIRS TEST- MAY 2023 SOLUTIONS

Q.74) Solution (b)

Statement Analysis:

Statement 1 Statement 2

Incorrect Correct

Kaundinya Wildlife Sanctuary is a wildlife Forest Type: Southern tropical dry deciduous
sanctuary and an elephant reserve situated forest, with patches of thorn, scrub, and
in Andhra Pradesh, India. grassy plains.
It is the only sanctuary in Andhra Pradesh Flora: Plants like Albizzia amara, Ficus
with a population of Asian elephants, which glomerata, Zizyphus xylocarpus,
migrated after 200 years from neighboring Gymnosporia Montana, etc; make this a
regions. green panoramic place with a breathtaking
This Sanctuary comes under Project Elephant floral diversity.
– a Countrywide Elephant Conservation Fauna: Insects like Gongylus praying mantis,
Project taken up by the Government of India. Painted grasshopper, Plain tiger butterfly,
Common grass yellow butterfly, etc; abound
this elephant land. Reptiles like Common
cobras, Rat snakes, Indian rock pythons,
Forest calotes, Skinks, etc; make this place
their abode. Birds like Partridges, quails,
storks, Cotton teal, etc; make this Sanctuary
their domicile. A further balance in the food
web is maintained by mammals like Indian
Elephants, panthers, Sloth bears, Wild boar,
Chowsingha, Nilgai, Hyena, Jackal, etc.
Source: https://www.thehindu.com/news/national/andhra-pradesh/jumbo-census-in-
seshachalam-hills-koundinya-wildlife-sanctuary-hints-at-sizeable-
presence/article66870607.ece

Q.75) Solution (d)

Statement Analysis:

Statement 1 Statement 2

Incorrect Incorrect

Article 163(1) of the Constitution says, “There The Deputy CM enjoys the same pay and
shall be a Council of Ministers with the Chief perks as a Cabinet Minister.

https://upscpdf.com/ https://upscpdf.com/ https://upscpdf.com/


https://upscpdf.com/ https://upscpdf.com/ https://upscpdf.com/

.
IASBABA’S PRELIMS CURRENT AFFAIRS TEST- MAY 2023 SOLUTIONS

Minister at the head to aid and advice Ten other states in the country (besides
the Governor in the exercise of his functions”. Karnataka) currently have Deputy CMs.
Article 164 (1) says “There shall be a Council Perhaps the first Deputy CM in India was
of Ministers with the Chief Minister at the Anugrah Narayan Sinha. He served as the
head to aid and advise the Governor in the Deputy Chief Minister cum Finance Minister
exercise of his functions.” from the time of the first Congress Ministry of
Hence, neither Article 163 nor Article 164 Bihar in 1937 till his death on 5 July 1957.
mentions a Deputy Chief Minister.
Source: https://indianexpress.com/article/explained/explained-politics/dk-shivakumar-
deputy-cm-post-explained-8621053/

Q.76) Solution (b)

Statement Analysis:

Statement 1 Statement 2

Incorrect Correct

African elephants are the largest land animals Elephants are matriarchal. It means they live
on Earth. in female-led groups. The matriarch is usually
The Asian elephant is the largest land the biggest and oldest.
mammal on the Asian continent.
They inhabit dry to wet forest and grassland
habitats in 13 range countries spanning South
and Southeast Asia.
They are slightly larger than Asian Elephants
and can be identified by their larger ears.
Asian elephants have smaller, rounded ears.
African elephants are keystone species, i.e.,
they play a critical role in their ecosystem.
They are also known as “ecosystem
engineers” as they shape their habitat in
many ways.
Statement 3

Correct

The Asian elephant is the largest land


mammal on the Asian continent.

https://upscpdf.com/ https://upscpdf.com/ https://upscpdf.com/


https://upscpdf.com/ https://upscpdf.com/ https://upscpdf.com/

.
IASBABA’S PRELIMS CURRENT AFFAIRS TEST- MAY 2023 SOLUTIONS

They inhabit dry to wet forest and grassland


habitats in 13 range countries spanning South
and Southeast Asia.
 IUCN Red List: Endangered
 CITES: Appendix I.
 Wildlife (Protection) Act, 1972:
Schedule I.
Source: https://indianexpress.com/article/india/finding-mates-for-indias-single-african-
jumbos-its-complicated-8621814/

Q.77) Solution (b)

Statement Analysis:

Statement 1 Statement 2

Incorrect Correct

Cold-bloodedness, also called Poikilothermy, This state distinguishes fishes, amphibians,


Ectothermy, or Heterothermy, reptiles, and invertebrate animals from
It is the state of having a variable body warm-blooded, or homoiothermic, animals
temperature that is usually only slightly (birds and mammals).
higher than the environmental temperature.
Because of their dependence upon
environmental warmth for metabolic
functioning, the distribution of terrestrial
cold-blooded animals is limited, with only a
few exceptions, to areas with a temperature
range of 5–10° to 35–40° C (41–50° to 95–
104° F). For cold-blooded animals living in the
arctic seas, temperatures may range from
below 0° C to 10–15° C (below 32° F to 50–59°
F).
Cold-blooded animals do maintain limited
control over internal temperature by
behavioral means, such as basking in sunlight
to warm their bodies.

https://upscpdf.com/ https://upscpdf.com/ https://upscpdf.com/


https://upscpdf.com/ https://upscpdf.com/ https://upscpdf.com/

.
IASBABA’S PRELIMS CURRENT AFFAIRS TEST- MAY 2023 SOLUTIONS

Source: https://www.thehindu.com/sci-tech/science/european-perch-larger-body-size-
warm-water/article66873491.ece

Q.78) Solution (a)


Statement Analysis:

Statement 1 Statement 2

Correct Incorrect

The International Pathogen Surveillance The International Pathogen Surveillance


Network (IPSN) is a global network Network (IPSN) will provide a platform to
launched by the World Health Organization connect countries and regions, improving
and its partners. systems for collecting and analyzing samples,
IPSN will help to protect people from using these data to drive public health
infectious disease threats through the power decision-making, and sharing that
of pathogen genomics. information more broadly.
IPSN will provide a platform to connect The IPSN, with a Secretariat hosted by the
countries and regions, improving systems for WHO Hub for Pandemic and Epidemic
collecting and analyzing samples, using these Intelligence, brings together experts
data to drive public health decision-making, worldwide at the cutting edge of genomics
and sharing that information more broadly. and data analytics, from governments,
philanthropic foundations, multilateral
organizations, civil society, academia, and the
private sector.

https://upscpdf.com/ https://upscpdf.com/ https://upscpdf.com/


https://upscpdf.com/ https://upscpdf.com/ https://upscpdf.com/

.
IASBABA’S PRELIMS CURRENT AFFAIRS TEST- MAY 2023 SOLUTIONS

All share a common goal: to detect and


respond to disease threats before they
become epidemics and pandemics, and to
optimize routine disease surveillance.
Source: https://www.who.int/news/item/20-05-2023-who-launches-global-network-to--
detect-and-prevent-infectious-disease-threats

Q.79) Solution (b)

Statement Analysis:

Statement 1 Statement 2

Correct Correct

Corporate Debt Market Development Fund It will be a backstop facility for the purchase
(CDMDF) is a form of an Alternative of investment-grade corporate debt
Investment Fund (AIF). securities in times of stress or in case of
The fund will have a corpus of ₹3,000 crores, market dislocation. The purpose of this fund
of which ₹2,700 crores will come from mutual is to bring about confidence in the corporate
fund debt schemes. The corporate bond bond market and boost secondary market
backstop fund will have 10x leverage and a liquidity.
sovereign guarantee. A backstop is an act of providing last-resort
The contribution will also be made by the support or security in a securities offering for
specified debt-oriented mutual fund schemes the unsubscribed portion of shares.
and asset management companies of mutual When a company is trying to raise capital
funds towards building the initial corpus of through an issuance, it may get a backstop
the CDMDF. from an underwriter or a major shareholder,
Access to the fund will be limited to specified such as an investment bank, to buy any of its
mutual fund schemes in proportion to the unsubscribed shares.
contribution made to the fund at a mutual
fund level. The SEBI board has approved the
framework for triggering CDMDF’s asset
purchases during market dislocation.
CDMDF, based on a guarantee to be provided
by the National Credit Guarantee Trust
Company (NCGTC) may raise funds, for the
purchase of corporate debt securities during
market dislocation.
The move will instill confidence among
corporate bond market participants and

https://upscpdf.com/ https://upscpdf.com/ https://upscpdf.com/


https://upscpdf.com/ https://upscpdf.com/ https://upscpdf.com/

.
IASBABA’S PRELIMS CURRENT AFFAIRS TEST- MAY 2023 SOLUTIONS

generally enhance secondary market


liquidity.
Source: https://www.thehindubusinessline.com/markets/new-backstop-fund-mooted-to-
bail-out-mf-debt-schemes-in-distress/article66676446.ece

Q.80) Solution (d)

Statement Analysis:

Statement 1 Statement 2

Correct Correct

The Institute of Chartered Accountants of The Institute of Chartered Accountants of


India is the only licensing cum regulating body India is a statutory body established under
of the financial audit and accountancy the Chartered Accountants Act, 1949 for the
profession in India. regulation of the profession of Chartered
The members of the Institute are known as Accountancy in India.
Chartered Accountants (CA). Becoming a
member requires passing the prescribed
examinations, three years of practical
training, and meeting other requirements
under the Act and Regulations.
Statement 3

Correct

It recommends the accounting standards be


followed by companies in India by National
Advisory Committee on Accounting Standards
(NACAS).
The institute has achieved recognition as a
premier accounting body in the fields of
education, professional development, and
maintenance of high accounting, auditing,
and ethical standards.
In India, accounting standards and auditing
standards are recommended by the National
Financial Reporting Authority (NFRA) to the
Government of India which sets the

https://upscpdf.com/ https://upscpdf.com/ https://upscpdf.com/


https://upscpdf.com/ https://upscpdf.com/ https://upscpdf.com/

.
IASBABA’S PRELIMS CURRENT AFFAIRS TEST- MAY 2023 SOLUTIONS

Standards on Auditing (SAs) to be followed in


the audit of financial statements in India.
Source: https://indianexpress.com/article/cities/delhi/delhi-hc-restrains-institute-cost-
accountants-india-icai-acronym-8523151/

Q.81) Solution (b)

Statement Analysis:

Statement 1 Statement 2

Incorrect Incorrect

Hydrogen produced from fossil fuels is Brown hydrogen is produced using coal
called grey hydrogen; this constitutes the where the emissions are released to the air.
bulk of the hydrogen produced today.
Statement 3 Statement 4

Correct Correct

Hydrogen generated entirely from renewable Hydrogen generated from fossil fuels with
power sources is called green hydrogen. carbon capture and storage options is
In the last process, electricity generated from called blue hydrogen.
renewable energy is used to split water into
hydrogen and oxygen.
Source: https://www.thehindu.com/news/cities/Delhi/how-gujarat-is-working-to-become-
indias-green-hydrogen-hub/article66871279.ece

Q.82) Solution (a)

Statement Analysis:

Statement 1 Statement 2

Correct Correct

Nyingmapa was founded by Kagyupa was founded by Tilopa (988-1069),


Padmasambhava, this is the oldest sect, and the Kagyupa tradition is headed by the
noted in the West for the teachings of the Karmapa Lama. Important Kagyupa teachers
Tibetan Book of the Dead. include Naropa, Marpa, and Milarepa.

https://upscpdf.com/ https://upscpdf.com/ https://upscpdf.com/


https://upscpdf.com/ https://upscpdf.com/ https://upscpdf.com/

.
IASBABA’S PRELIMS CURRENT AFFAIRS TEST- MAY 2023 SOLUTIONS

Statement 3 Statement 4

Incorrect Incorrect

Sakyapa was founded by Gonchok Gyelpo Gelugpa (The Virtuous School) was founded
(1034-1102) and his son Gunga Nyingpo by Tsong Khapa Lobsang Drakpa (also called
(1092-1158). Je Rinpoche) (1357 – 1419), this tradition is
headed by the Dalai Lama.
Source: https://theprint.in/world/indias-pledge-to-revive-forgotten-buddhist-folklore-
across-borders/1587170/

Q.83) Solution (b)

Statement Analysis:

Statement 1 Statement 2

Incorrect Correct

National AYUSH Mission is a Centrally The central government provides a 60% share
Sponsored Scheme under the Ministry of and the state government provides a 40%
AYUSH. share for its funding.
The government of India launched National
AYUSH Mission (NAM) in 2014. Components of the Mission
The aims of NAM are to mainstream AYUSH Mandatory Components
Systems into health care services, to develop  AYUSH Services.
evidence-based AYUSH management  AYUSH Educational Institutions.
protocol through scientific documentation,  Quality Control of ASU&H (Ayurveda,
and to ensure the accessibility of quality Siddha, and Unani & Homoeopathy)
AYUSH services. Drugs.
 Medicinal Plants.

Flexible Component
 AYUSH Wellness Centres comprising
Yoga and Naturopathy
 Tele-medicine
 Innovations in AYUSH including Public
Private Partnership
 IEC (Information, Education, and
Communication) activities

https://upscpdf.com/ https://upscpdf.com/ https://upscpdf.com/


https://upscpdf.com/ https://upscpdf.com/ https://upscpdf.com/

.
IASBABA’S PRELIMS CURRENT AFFAIRS TEST- MAY 2023 SOLUTIONS

Voluntary certification scheme:


Project-based, etc.
Source: https://pib.gov.in/PressReleasePage.aspx?PRID=1925182

Q.84) Solution (c)

Statement Analysis:

Statement 1 Statement 2

Correct Incorrect

Green deposits are interest-bearing deposits All scheduled commercial banks including
offered by banks and NBFCs for a fixed period, small finance banks (excluding RRBs, LABs,
with proceeds used to promote sustainable and payment banks) and all deposit-taking
investments in environmentally-friendly NBFCs registered with RBI, including HFCs, are
sectors. eligible to accept green deposits.
They include sectors such as Renewable
Energy, Energy Efficiency, Clean
Transportation, Climate Change Adaptation,
Sustainable Water and Waste Management,
Green Buildings, etc.
Projects involving new or existing extraction,
production, and distribution of fossil fuels;
Nuclear power generation; Direct waste
incineration; Landfill projects; Hydropower
plants larger than 25 MW, etc are excluded
from green deposits.
Statement 3

Correct

Green deposits are accepted in Indian Rupees


denominations only.
Green deposit funds are subject
to independent third-party verification on an
annual basis.
Source: https://www.thehindu.com/opinion/op-ed/explained-what-are-rbi-regulations-on-
green-deposits/article66866265.ece

https://upscpdf.com/ https://upscpdf.com/ https://upscpdf.com/


https://upscpdf.com/ https://upscpdf.com/ https://upscpdf.com/

.
IASBABA’S PRELIMS CURRENT AFFAIRS TEST- MAY 2023 SOLUTIONS

Q.85) Solution (c)

Statement Analysis:

Statement 1 Statement 2

Correct Correct

The IREDA is a Public Limited Government Indian Renewable Energy Development


Company established as a Non-Banking Agency Limited (IREDA) is a Mini Ratna
Financial Institution in 1987 engaged in (Category – I) Government of India Enterprise
promoting, developing, and extending under the administrative control of the
financial assistance for setting up projects Ministry of New and Renewable Energy
relating to new and renewable sources of (MNRE).
energy and energy efficiency/conservation. The Indian Renewable Energy Development
IREDA’s long-term objective is to contribute Agency (IREDA) offers a credit enhancement
to the development of a bond market for guarantee scheme to support the issuance of
renewable energy projects in India. bonds by wind and solar energy project
developers.
By providing unconditional and irrevocable
partial credit guarantees, IREDA aims to
enhance the credit rating of bonds for
renewable energy projects, thereby
improving their marketability and liquidity,
and attracting lower-cost and longer-term
funding for project developers. The amount
raised by credit-enhanced bonds shall only
serve to repay existing debt partially or fully.
Source: https://www.pib.gov.in/PressReleasePage.aspx?PRID=1925552

Q.86) Solution (b)

Statement Analysis:

Statement 1 Statement 2

Incorrect Correct

A global depositary receipt is a type of bank GDRs are commonly used by issuers to raise
certificate that represents shares of stock in capital from international investors through
an international company. private placement or public stock offerings.

https://upscpdf.com/ https://upscpdf.com/ https://upscpdf.com/


https://upscpdf.com/ https://upscpdf.com/ https://upscpdf.com/

.
IASBABA’S PRELIMS CURRENT AFFAIRS TEST- MAY 2023 SOLUTIONS

The shares underlying the GDR remain on A global depositary receipt is very similar to
deposit with a depositary bank or custodial an American depositary receipt (ADR) except
institution. that an ADR only lists shares of a foreign
company in U.S. markets.
GDRs make it possible for a company (the
issuer) to access investors in capital markets
beyond the borders of its own country.
Note:

 American Depository Receipts is a form of equity security that was created specifically
to simplify foreign investing for American investors.
 An ADR is issued by an American bank or broker.
 It represents one or more shares of foreign-company stock held by that bank in the
home stock market of the foreign company.
 Indian Depository Receipts (IDR) are in Indian rupees and are created by a domestic
depository (custodian of securities registered with SEBI (Securities and Exchange
Board of India).
 It is issued against the underlying equity of the company to enable foreign companies
to raise funds from the Indian securities Markets.
Source: https://www.thehindubusinessline.com/markets/tata-consumer-products-to-delist-
gdrs-from-june-23/article66858443.ece

Q.87) Solution (a)

Statement Analysis:

Statement 1 Statement 2

Correct Incorrect

‘Operation Dhvast’ focuses on the terrorist- ‘Operation Dhvast’ was launched by National
gangster-drug smugglers’ network cases. Investigation Agency across eight states.
The focus of the raids was on weapon
suppliers, financiers, logistic providers, and
hawala operators associated with the
hardcore gangs working with drug smugglers
and terrorists based out of other countries
like Pakistan and Canada.
NIA investigations have revealed that the
conspiracies were being hatched in jails of
different states and were being executed by

https://upscpdf.com/ https://upscpdf.com/ https://upscpdf.com/


https://upscpdf.com/ https://upscpdf.com/ https://upscpdf.com/

.
IASBABA’S PRELIMS CURRENT AFFAIRS TEST- MAY 2023 SOLUTIONS

an organised network of operators based


abroad.
The spotlight on these gangs has become
sharper after reports of several prisons
becoming havens of the deadly nexus and
hubs of gang wars, which recently resulted in
violence and murder inside Goindwal Jail and
Tihar Jail
Source: https://www.indiatoday.in/india/story/operation-dhwast-nia-mega-crackdown-8-
states-gangster-and-terrorist-2380611-2023-05-17

Q.88) Solution (c)

Statement Analysis:

Statement 1 Statement 2

Correct Incorrect

European Bank for Reconstruction and EBRD is headquartered in London, United


Development (EBRD) is a multilateral Kingdom.
financial institution established in 1991.
The organization was developed to help
Eastern European and ex-Soviet countries
transition into democracies by developing
free-market economies after the fall of
communism.
It is publicly owned by shareholders in 69
countries.
Statement 3

Correct

It only supports countries that are committed


to democratic principles
The bank finances large and small projects in
a variety of sectors including public works,
agribusiness, natural resources, and
municipal infrastructure.
The organization does not finance projects
related to the tobacco industry, defense,

https://upscpdf.com/ https://upscpdf.com/ https://upscpdf.com/


https://upscpdf.com/ https://upscpdf.com/ https://upscpdf.com/

.
IASBABA’S PRELIMS CURRENT AFFAIRS TEST- MAY 2023 SOLUTIONS

certain alcoholic products, stand-alone


gambling facilities, or substances banned by
international law.
Despite its public sector shareholders, it
invests mainly in private enterprises, together
with commercial partners.
Note: India has become the 69th shareholder of the European Bank for Reconstruction and
Development (EBRD). This move will enable Indian companies to undertake joint investments
in regions in which the EBRD operates.
Source: https://theprint.in/world/ebrd-pledges-more-support-for-ukraine-eyes-capital-
boost/1581166/

Q.89) Solution (d)

Statement Analysis:

 Snow leopards are distributed across the 100,146 sq. km of snowy forests in five
Himalayan states – Jammu & Kashmir, Uttarakhand, Himachal Pradesh, Sikkim &
Arunachal Pradesh.
 Hemis National Park, Gangotri National Park, Khangchendzonga National Park, and
Great Himalayan National Park are some protected areas where snow leopards are
known to be found.
 It is listed as Vulnerable on the IUCN Red List because the global population is
estimated to number fewer than 10,000 mature individuals and is expected to decline
by about 10% by 2040.
National Parks/Wildlife State
Sanctuaries
1. Kibber Wildlife Sanctuary Himachal Pradesh
2. Ulley Valley Ladakh
3. Hemis National Park Himachal Pradesh

4. Kishtwar National Park Jammu and Kashmir


Source: https://indianexpress.com/article/cities/jammu/in-remote-jk-national-park-camera-
traps-snap-elusive-snow-leopard-read-out-a-success-story-8615362/

https://upscpdf.com/ https://upscpdf.com/ https://upscpdf.com/


https://upscpdf.com/ https://upscpdf.com/ https://upscpdf.com/

.
IASBABA’S PRELIMS CURRENT AFFAIRS TEST- MAY 2023 SOLUTIONS

Q.90) Solution (c)

Statement Analysis:

Statement 1 Statement 2

Correct Correct

The National Medical Commission Act of There are four boards in the National Medical
2019 establishes the National Medical Commission
Commission (NMC). Hence it is a statutory  Under-Graduate Medical Education
body. Board (UGMEB)- sets norms for
It is responsible for the creation and undergraduate courses
regulation of all elements of medical  Post-Graduate Medical Education
education, practice, and institutions. Board (PGMEB)- sets norms for post-
National Medical Commission consists of 25 graduate courses
members including  Medical Assessment and Rating
The Chairperson, Presidents of Postgraduate Board– inspects and rates the medical
Medical Education Boards, and Presidents of education institutes
Undergraduate Medical Education Boards  Ethics and Medical Registration
Director General of Indian Council of Medical Board– regulates the professional
Research (ICMR) conduct of the doctors and registers
Director General of Health Services. them
Source: https://www.newindianexpress.com/nation/2023/may/18/doctors-peeved-at-nmc-
notificationon-licence-transfer-2576217.html

Q.91) Solution (a)

Statement Analysis:

Statement 1 Statement 2

Correct Incorrect

The Production Linked Incentive Scheme’s It was launched in April 2020, for the Large
objective is to make domestic manufacturing Scale Electronics Manufacturing sector, but
globally competitive and to create global later towards the end of 2020 was introduced
Champions in manufacturing. for 10 other sectors. This scheme was
The strategy behind the scheme is to introduced in line with India’s Atmanirbhar
offer companies incentives on incremental Bharat Campaign.
sales from products manufactured in India,
over the base year. They have been

https://upscpdf.com/ https://upscpdf.com/ https://upscpdf.com/


https://upscpdf.com/ https://upscpdf.com/ https://upscpdf.com/

.
IASBABA’S PRELIMS CURRENT AFFAIRS TEST- MAY 2023 SOLUTIONS

specifically designed to boost domestic


manufacturing in sunrise and strategic
sectors
It intends to curb cheaper imports and reduce
import bills, improve the cost
competitiveness of domestically
manufactured goods, and enhance domestic
capacity and exports.
Source: https://indianexpress.com/article/india/cabinet-rs-17000-cr-it-hardware-pli-
scheme-2-0-8614534/

Q.92) Solution (b)

Statement Analysis:

Statement 1 Statement 2

Incorrect Correct

The Department of Telecommunications It enables people across India to track and


(DoT) has launched the Sanchar Saathi Portal. block their lost or stolen mobile phones.
Through this portal, people can also verify the
authenticity of used devices before making a
purchase, and track how many numbers are
associated with their identity without their
knowledge.
Key sections of the portal include the Central
Equipment Identity Register (CEIR) regarding
blocking and tracking of phones, the ‘Know
Your Mobile’ feature that allows users to
verify the authenticity of second-hand mobile
phones, and the TAFCOP (Telecom Analytics
for Fraud Management and Consumer
Protection) facility to check numbers
associated with an identity.
Source: https://www.financialexpress.com/industry/lost-your-phone-a-portal-to-help-track-
your-device/3090403/

https://upscpdf.com/ https://upscpdf.com/ https://upscpdf.com/


https://upscpdf.com/ https://upscpdf.com/ https://upscpdf.com/

.
IASBABA’S PRELIMS CURRENT AFFAIRS TEST- MAY 2023 SOLUTIONS

bQ.93) Solution (d)

Statement Analysis:

 A circular economy reduces material use, redesigns materials, products, and services
to be less resource intensive, and recaptures “waste” as a resource to manufacture
new materials and products.

 A circular economy is based on three clear principles that bring multiple benefits,
including benefits for our climate. These are:
 Eliminating waste and pollution, in turn, reduces and avoids emissions across the
value chain.
 Circulating products and materials, in turn, enable embodied emissions to be retained.
 Regenerating nature, thereby also improving carbon sequestration.
Source: https://www.financialexpress.com/industry/how-circular-economy-is-
revolutionising-plastics-packaging-industry/3092329/

Q.94) Solution (b)

Statement Analysis:

Statement 1 Statement 2

Incorrect Correct

Trimbakeshwar Mahadev Temple is in Trimbakeshwar is the abode of one of


Trimbakeshwar town, an ancient Hindu the twelve Jyotirlingas. The extraordinary
Pilgrim centre located at the source of feature of the Jyotirlinga located here is
the Godavari River, the longest river in the Linga in the temple is in the form of a
peninsular India. three-faced embodying Tridev, Lord Brahma,
There are entry gates on all four sides, viz. Lord Vishnu, and Lord Shiva.
East, West, South, and North. As per spiritual
conceptions the direction East denotes the
beginning, the West denotes maturity, the
South denotes fulfillment or completion and
the North stands for revelation.
In the year 1954, the Sansthan was registered
under the public trust registration act. Shri
Trimbakeshwar Sansthan takes care of daily
three-time pooja, all the festivals, and gala
carnivals of the year.

https://upscpdf.com/ https://upscpdf.com/ https://upscpdf.com/


https://upscpdf.com/ https://upscpdf.com/ https://upscpdf.com/

.
IASBABA’S PRELIMS CURRENT AFFAIRS TEST- MAY 2023 SOLUTIONS

Statement 3

Correct

The present Trimbakeshwar temple was


constructed by the third Peshwa Balaji
Bajirao (1740-1760) on the site of an old
temple.
Source: https://indianexpress.com/article/cities/mumbai/nashik-temple-as-fadnavis-orders-
sit-probe-into-forcible-entry-claims-of-decades-old-ritual-8613281/lite/

Q.95) Solution (a)

Statement Analysis:

Statement 1 Statement 2

Correct Incorrect

Gekko Mizoramensis is arboreal, meaning it The species is found in Southeast Asia. It is


primarily inhabits trees. It is nocturnal, being specifically found in Mizoram, a region in
most active during the night. The gecko has India, as well as parts of Bangladesh,
the ability to glide from one tree to another, Myanmar, Thailand, and Cambodia.
aided by its webbed limbs and flat tail.
However, it does not possess the capability to
fly.
Source: https://www.thehindu.com/sci-tech/science/mizoram-records-new-parachute-
gecko/article66857869.ece

Q.96) Solution (b)

Statement Analysis:

Statement 1 Statement 2

Incorrect Correct

LIBOR, the acronym for London Interbank It is used for pricing of interest rate swaps,
Offer Rate, is the global reference rate currency rate swaps as well as mortgages.
for unsecured short-term borrowing in the
interbank market.

https://upscpdf.com/ https://upscpdf.com/ https://upscpdf.com/


https://upscpdf.com/ https://upscpdf.com/ https://upscpdf.com/

.
IASBABA’S PRELIMS CURRENT AFFAIRS TEST- MAY 2023 SOLUTIONS

It acts as a benchmark for short-term interest It is an indicator of the health of the financial
rates. system and provides an idea of the trajectory
The five currencies for which LIBOR is of impending policy rates of central banks.
computed are the Swiss Franc, Euro, Pound LIBOR is administered by the Intercontinental
Sterling, Japanese Yen, and US Dollar. Exchange or ICE. It is computed for five
currencies with seven different maturities
ranging from overnight to a year.
Source: https://indianexpress.com/article/business/banking-and-finance/rbi-tells-banks-to-
complete-transition-from-libor-by-july-8606542/

Q.97) Solution (d)

Statement Analysis:

Statement 1 Statement 2

Correct Correct

The Khasis are one of the three Under the Constitution of India, the Khasis
indigenous matrilineal communities in the have been granted the status of a Scheduled
northeastern State. The other two are Garos Tribe.
and Jaintias. They are among the few Austroasiatic-
The Khasis inhabit the eastern part of speaking peoples in South Asia.
Meghalaya, in the Khasi and Jaintia Hills. The Khasis, the Jaintias, and the Garos have
Khasis residing in Jaintia hills are now better a matrilineal society.
known as Jaintias. Earlier in the conservative Jaintia non-
They are also called Christian families, however, the father only
The Khasis occupying the northern lowlands visits the family at night and is not responsible
and foothills are generally called Bhois. for the maintenance of the family.
The Khasi people are an indigenous ethnic
group of Meghalaya in north-eastern India
with a significant population in the bordering
state of Assam and certain parts of
Bangladesh.
The Khasi people form the majority of the
population of the eastern part of Meghalaya,
and are the state’s largest community, with
around 48% of the population of Meghalaya.
Statement 3

https://upscpdf.com/ https://upscpdf.com/ https://upscpdf.com/


https://upscpdf.com/ https://upscpdf.com/ https://upscpdf.com/

.
IASBABA’S PRELIMS CURRENT AFFAIRS TEST- MAY 2023 SOLUTIONS

Correct

Khasis follow a matrilineal system of


inheritance.
In the Khasi society, it is only the youngest
daughter, or “KaKhadduh” who is eligible
to inherit the ancestral property.
If ‘KaKhadduh’ dies without any daughter
surviving her, her next elder sister inherits the
ancestral property, and after her, the
youngest daughter of that sister.
Failing all daughters and their female issues,
the property goes back to the mother’s sister,
the mother’s sister’s daughter, and so on.
The KaKhadduh’s property is actually the
ancestral property and so if she wants to
dispose of it, she must obtain the consent and
approval of the uncles and brothers.
Source: https://www.thehindu.com/news/national/other-states/row-over-adopting-fathers-
surname-in-matrilineal-meghalaya/article66856681.ece

Q.98) Solution (c)

Statement Analysis:

Statement 1 Statement 2

Correct Correct

The ‘Adopt a Heritage: Apni Dharohar, Apni Private firms, corporates, NGOs, and public
Pehchaan’ scheme is an initiative of the sector units can enter into agreements with
Ministry of Tourism, in collaboration with the Union Ministry of Culture to adopt and
the Ministry of Culture and maintain State-owned archaeological sites or
the Archaeological Survey of India along with monuments.
the State/UT governments. It aims to develop synergy among all partners
The project primarily focuses on providing to effectively promote ‘responsible and
basic amenities like cleanliness, public sustainable tourism’.
conveniences, drinking water, ease of access Agencies/Companies would become
for differently abled and senior citizens, ‘Monument Mitras’ through the innovative
and enhanced facilities like surveillance concept of ‘Vision Bidding’, where the agency
with the best vision for the heritage site will

https://upscpdf.com/ https://upscpdf.com/ https://upscpdf.com/


https://upscpdf.com/ https://upscpdf.com/ https://upscpdf.com/

.
IASBABA’S PRELIMS CURRENT AFFAIRS TEST- MAY 2023 SOLUTIONS

systems, night viewing facilities, etc., to be allowed to maintain the monument


provide an enhanced tourism experience. through their CSR activities.
The project began with 93 ASI monuments
and has extended to heritage, natural, and
tourist sites across India, some of which are
not protected under ASI currently.
Source: https://www.newindianexpress.com/states/odisha/2023/may/15/adopt-a-heritage-
scheme-by-next-month-gk-reddy-2575211.html

Q.99) Solution (b)

Statement Analysis:

Statement 1 Statement 2

Incorrect Correct

The National Gallery of Modern Art (NGMA) is It is run and administered as a subordinate
the premier art gallery that aims to acquire office to the Ministry of Culture, Government
and preserve works of modern art from the of India.
1850s onward. The main museum in New Delhi was
Its collection includes works by artists such as established in 1954 by the Government of
Raja Ravi Verma, Rabindranath Tagore, India.
Abanindranath Tagore, Gaganendranath NGMA has two other branches, one in
Tagore, Nandalal Bose, Jamini Roy, Amrita Mumbai and another in Bangalore.
Sher-Gil as well as foreign artists like Thomas
Daniell.
Source: https://pib.gov.in/PressReleseDetailm.aspx?PRID=1924012

Q.100) Solution (b)

Statement Analysis:

Statement 1 Statement 2

Incorrect Correct

The Sahyadri Tiger Reserve is located in the  Vegetation: It comprises rich


Sahyadri Ranges of Western Ghats in evergreen, semi-evergreen, and moist
Maharashtra. deciduous forests.

https://upscpdf.com/ https://upscpdf.com/ https://upscpdf.com/


https://upscpdf.com/ https://upscpdf.com/ https://upscpdf.com/

.
IASBABA’S PRELIMS CURRENT AFFAIRS TEST- MAY 2023 SOLUTIONS

These range from a common boundary The habitat of Sahyadri is composed of


between Maharashtra, Karnataka, and Goa. woodlands, grasslands, and plateaus.
It is the first Tiger Reserve of Western  The most distinct feature of the Tiger
Maharashtra and the fourth Tiger Reserve of Reserve is the presence of numerous
the State spreading over two Protected Areas barren rocky and lateritic plateaus,
of Koyana Sanctuary (KWLS) and Chandoli locally called “Sadas”, with less
National Park (CNP). perennial vegetation and overhanging
KWLS and CNP are part of the UNESCO World cliffs on the edges along with
Natural Heritage Sites. Birdlife International numerous fallen boulders with dense
has declared this area an ‘Important Bird thorny bushes.
Area’.  Fauna: The Tiger Reserve is home to
CNP has a number of ecotone areas and is the endangered species of top
thus highly species-rich. carnivores such as the Tiger, Wild dog,
and Leopard. As many as 30 species of
mammals have been recorded. In
addition, the area is home to Vultures
and Crocodiles. The habitat also
supports Hornbills and many other
endemic Birds. The Blue finned
Mahasheer fish is present in the
Koyana waters.
Source: https://timesofindia.indiatimes.com/city/kolhapur/374-animals-recorded-in-
wildlife-census-at-sahyadri-tiger-reserve/articleshow/100238643.cms?from=mdr

https://upscpdf.com/ https://upscpdf.com/ https://upscpdf.com/

You might also like